Nothing Special   »   [go: up one dir, main page]

Внутрен Болезни -2 24.12.20

Download as docx, pdf, or txt
Download as docx, pdf, or txt
You are on page 1of 97

#1

!The student turned to the clinic complaining of a dry, painful cough, pain behind the sternum. By
auscultation - dry, scattered rales. He took mucaltin on his own for 5 days without effect.
The appointment of which of the following groups of drugs is MOST appropriate?
*Anticholinergics
*Antileukotriene preparations
*Mukalitiki
* +b-2 short-acting agonists
*Antihistamines

#2
!A 19-year-old student diagnosed with «Acute bronchitis» received outpatient treatment for 7 days.
There are no complaints at the repeated reception, the condition is satisfactory. By auscultation is
vesicular breathing, no wheezing.
What is the most appropriate tactic for managing a patient with acute bronchitis?
*Administration of long-acting b-2 agonists
*Prescribing antibiotics of another series
*Nebulizer therapy with corticosteroid
* +Spa treatment
*Administration of short-acting b-2 agonists

#3
!A 19-years-old student after an acute respiratory illness has developed a dry, painful cough, pain
behind his sternum. During auscultation in the lungs, hard breathing, single dry rales are heard. In
the blood test: white blood cells 5.0 x 10 9 / l, ESR 14 mm / hour. Diagnosed acute bronchitis.
Which of the following non-drug therapy methods is MOST appropriate?
*Physical education
*+ Physiotherapy
*Remediation of foci of infection
*Dieting
*Hardening the body

#4
!A 30 years-old woman is diagnosed with «SARS. Acute bronchitis, obstructive syndrome». What
is the most appropriate medication must be prescribed?
*Azithromycin
*Bromhexine
*Ambroxol
*Pulmicort
* +Salbutamol

#5
!A 19 years-old student with a diagnosis of «Acute bronchitis» received outputient treatment for 7
days. There are no complaints at the repeated reception, the condition is satisfactory. By
auscultation - vesicular breathing, no wheezing.
What is the most appropriate tactic for managing a patient with acute bronchitis?
*Administration of long-acting b-2 agonists
*Prescribing antibiotics of another series
*Nebulizer therapy with corticosteroid
*+ Spa treatment
*Administration of short-acting b-2 agonists

#6
!A 48-year-old male complains of a cough with mucopurulent sputum for a long time, an increase
in body temperature up to 37.8 ° C, weakness, malaise that appeared after hypothermia.
Objectively: cyanosis of the nasolabial triangle. By auscultation - there is hard breathing.
Preliminary diagnosis: Chronic bronchi- tis, exacerbation.
Which of the following groups of drugs is MOST appropriate?
*Bronchodilators
* +Antibiotics
*Antispasmodics
*Anti-aggregates
*Corticosteroids

#7
!A 40-year-old female complains of a cough with mucopurulent sputum for a long time, an
increase in body temperature up to 37.5 ° C, weakness, and malaise. From the anamnesis smokes
for 12 years. Aller- gic rhinitis last 5 years. Objectively: pallor of the skin. Auscultation - hard
breathing with moist rales that disappear after coughing. Diagnosed with chronic bronchitis,
exacerbation.
What drugs are most appropriate to prescribe?
*Bronchodilators
*Vitamins
*Antispasmodics
* +Antibiotics
*Corticosteroids

#8
!A 45-year-old man complains of a cough with mucopurulent sputum, an increase in body
temperature up to 37.8 ° C, weakness, and malaise. By auscultation there is hard breathing with dry
rales. Preliminary diagnosis: Chronic bronchitis, exacerbation.
What drugs are most appropriate?
*b-2 long-acting agonists
* +Antibacterial drugs
*Inhaled corticosteroids
*Mucolitics
*b-2 short-acting agonists

#9
!A 38-year-old woman complains of coughing with mucopurulent sputum for a long time,
shortness of breath, weakness, and malaise. From the anamnesis smokes for 12 years. Objectively:
pallor of the skin. By auscultation there is hard breathing, dry wheezing during forced expiration.
Which drug is MOST advisable to appoint for bronchodilator?
*Mukaltin
*+ Salbutamol
*Ambroxol
*Amoxicillin
*Fluoroquinolones

#10
!A 44-year-old man was diagnosed with Chronic bronchitis, exacerbation. He receives outpatient
treat- ment with antibiotics and mucolytics for 5 days. The condition is closer to satisfactory, body
temperature up to 36.70С. By auscultation there is hard breathing with single dry rales.
What is the most appropriate medication?
*Azithromycin
*Cepepime
*Ambroxol
*Pulmicort
* +Salbutamol
#11
!A 35-year-old woman complains of a cough with mucopurulent sputum, an increase in body
temperature to 37.8 ° C, weakness, and malaise. By auscultation there is hard breathing.
Preliminary diagnosis: Chronic bronchitis, exacerbation. What drugs are most appropriate?
*b-2 long-acting agonists
* +Antibacterial drugs
*Inhaled corticosteroids
*Mukolytics
*b-2 short-acting agonists

#12
!A 45-year-old man is diagnosed with community-acquired pneumonia. What drugs are most
appropriate to prescribe?
*Bronchodilators
*Vitamins
*Antispasmodics
* +Antibiotics
*Corticosteroids

#13
!A 30 years old female has been diagnosed with Community-acquired pneumonia of the lower lobe
of the right lung. Which drug is the MOST appropriate to prescribe?
*Salbutamol
*+ Amoxiclav
*prednisone
*Seretide
*Eufillin

#14
!The third day you observe a 35-year-old patient with a diagnosis of Community-acquired
pneumonia. He receives antibiotic therapy (penicillin series). Despite this patient’s condition of
health does not improve, body temperature has not returned to normal.
What is the most appropriate patient’s management tactic?
*+ Change to cephalosporin antibiotics
*Increase the dose of the resulting antibiotic
*Add to the treatment of bronchodilators
*Continue treatment with the selected antibiotic
*Prescribe inhaled corticosteroids

#15
!A 23 years old woman is diagnosed with community-acquired pneumonia of the lower lobe of the
right lung.
Which of the following is the most appropriate?
*Salbutamol
*+ Amoxiclav
*prednisone
*Seretide
*Eufillin

#16
!A 32 years old man. He went to the hospital with complaints of an asthma attack, shortness of
breath. From the anamnesis: similar attacks have been observed before, but stopped on their own.
This attack developed suddenly during the repair of the apartment. Smokes 1 pack per day.
Objectively: the skin is moist, moderate cyanosis of the lips. By auscultation of the lungs,
breathing is weakened, dry wheezing. Which of the following is the MOST acceptable for this
situation?
* +Short-acting bronchodilators
*Long-acting bronchodilators
*Antibacterial drugs
*expectorant drugs
*Inhaled glucocorticosteroids

#17
!A 25-year-old woman was admitted to the admission department with complaints of severe
shortness of breath with difficulty exhaling, reaching suffocation, poorly stopped by taking
salbutamol. From the an- amnesis of the disease: complains are for about 2 years, when she began
to notice the appearance of shortness of breath with difficulty exhaling during walks, especially in
the spring. She went to the clinic and was diagnosed with bronchial asthma.
Which of the following groups of drugs is MOST acceptable for basic therapy?
*Antibacterial therapy
* +Inhaled glucocorticosteroids
*Short-acting bronchodilators
*detoxification therapy
*Long-acting bronchodilators

#18
!A 32 years old man. He went to the admission department with complaints of an asthma attack,
shortness of breath. From the anamnesis: similar attacks have been observed before, but stopped on
their own. This attack developed suddenly during the repair of the apartment. Smokes 1 pack per
day. Objectively: the skin is moist, moderate cyanosis of the lips. By auscultation of the lungs,
breathing is weakened, dry wheezing.
Which of the following methods of drug administration is the MOST acceptable?
*+ Inhalation
*Sublingual
*Intramuscular
*infusion
*Oral

#19
!A 45-year-old man is diagnosed with «Chronic Obstructive Pulmonary Disease, moderate». The
appointment of which of the following groups of drugs is MOST appropriate?
*+ Bronchodilators
*Vitamins
*Antispasmodics
*Anti-aggregates
*Antibiotics

#20
!A 40 years old man, smokes 1 pack per day. Complaints of cough with mucous sputum in the
morning. During spirometry: FEV1-80%, Tiffno index-69%. Diagnosed with COPD, mild severity.
What is the most appropriate patient’s management tactic?
* +Avoid smoking
*β2 - short-acting agonists
*β2 - long-acting agonists
*M-anticholinergics
*Inhaled corticosteroids

#21
!A 45 years old man, smokes for 6 years at 0.5 packs per day. Smoker Index 3. Diagnosed with
Chronic Obstructive Pulmonary Disease, Mild, Remission.
According to the clinical protocol "Chronic obstructive pulmonary disease", MH RK 2016 What is
the most appropriate patient’s management tactic?
* +Avoid smoking
*Antibacterial therapy
*Surgical treatment
*Therapeutic exercise
*Oxygen therapy
#22
!A 40-year-old man smokes 1 pack a day for 10 years. Complaints of cough with mucous sputum
in the morning. During spirometry: FEV1-80%, Tiffno index-69%. Diagnosed with COPD, mild
severity.
What is the most appropriate patient’s management tactic?
*Antibiotics
*+ β2 - short-acting agonists
*β2 - long-acting agonists
*M-anticholinergics
*Inhaled corticosteroids

#23
!A 67-year-old man is registered for COPD. He complained of a cough with mucopurulent sputum,
in- creased shortness of breath during physical exertion, an increase in body temperature to 38 ° C.
Patient associates his condition with hypothermia. During auscultation against the background of
weakened breathing with an extended expiration, scattered dry rales from two sides are heard.
Which of the following is the most appropriate?
*Salbutamol
*+ Amoxiclav
*prednisone
*Theof Illin
*Amikacin

#24
!Which of the following methods of non-drug treatment of respiratory failure is the MOST
effective?
*Mustard plasters on the back
*Hot foot baths
*Acupuncture
*+ Oxygen therapy
*Herbal medicine

#25
!A 25-year-old man with a diagnosis of «Community-acquired pneumonia of the lower lobe of the
right lung» received outpatient treatment. On the 7th day there are no complaints, the condition is
satisfactory, with auscultation vesicular breathing, no wheezing.
What further management tactics of the patient is MOST appropriate?
*Continue antibiotic therapy
*Prescribe antibiotics of a different row
*Proceed to work
*+ Control X-ray of chest on the 10th day
*Prescribe inhaled corticosteroids

#26
!A 50 years old woman. Complaints of severe pain in the right side of the chest, aggravated by
inhalation, fever, severe weakness, dry cough. Complains appeared after hypothermia. Objectively:
the chest is of the usual form, in the lower lobe of the right lung a dull percussion sound. With
auscultation in the lower parts of the right lung, breathing is sharply weakened.
Which of the following diagnoses is MOST characteristic for this chest X-ray?
*Community-acquired lower-lobe pneumonia
*Aspiration pneumonia on the right
*Dry pleurisy on the right
*+ Exudative pleurisy on the right
*Nosocomial upper lobar pneumonia

#27
!A 40-year-old man with complaints of chest pain with a deep breath, shortness of breath, fever,
severe weakness, dry cough. Objectively: the chest is of the usual form, in the lower lobe of the left
lung a dull percussion sound. With auscultation in the lower parts of the left lung, breathing is
sharply weakened. Which of the following diagnoses is MOST characteristic for this chest
radiograph?

*Community-acquired lower-lobe pneumonia


*Aspiration pneumonia on the right
*Dry pleurisy on the right
*+ Exudative pleurisy on the left
*Nosocomial upper lobar pneumonia

#28
!Analysis of pleural fluid: specific gravity - 1020, color - cloudy, protein - 30g / l, Rivalt reaction
+; mi- croscopy: white blood cells are in sight, atypical cells are not, mycobacterium tuberculosis is
not.
Which of the following diagnoses is MOST probable by the result of pleural fluid analysis?
*Dry pleurisy
*+ Exudative pleurisy
*Tuberculous pleurisy
*Pleurisy with heart failure
*Tumor pleurisy #29
!A 55 years old woman, cooker. Complaints of cough with mucous sputum in the morning,
shortness of breath during physical exertion. Spirometry indices: FEV1-65%, Tiffno index-60%.
Diagnosed with COPD, moderate.
What is the most appropriate medication?
*Seretide
* +Spiriva
*Symbicort
*Salbutamol
*Ceftriaxone

#30
!The 19 years old student after acute respiratory disease on the 3rd day, a cough appeared with a
mucopu- rulent spurum.
Which of the following non-drug therapy methods is MOST aimed at facilitating sputum
production?
* +Vibration massage
*Physical education
*Remediation of foci of infection
*Physiotherapy
*Hardening the body

#31
!A 49-year-old man is worried about severe expiratory dyspnea, a paroxysmal cough with sputum.
Smokes for 20 years. On examination: a barrel-shaped chest, percussion - box sound, auscultation -
weakened vesicular breathing, shortened exhalation, rare dry rales.
Which of the following drugs is most appropriate?
*+ Ipratropium bromide
*Sodium cromoglycate
*Amoxicillin / clavulanate
*Budesonide
*Beclomethasone

#32
!A 49-year-old man complains of shortness of breath that occurs during physical exertion,
paroxysmal cough with sputum. Smoked for about 20 years. Objectively: the skin is dry, moderate
“gray” cyanosis, the nails are changed according to the type of “watch glasses”, the chest is
swollen with emphysema. X- ray: decreased transparency of the lungs, depleted pulmonary pattern.
FEV1 = 65%.
What is the most appropriate treatment tactic?
* +Ipratropium bromide
*Sodium cromoglycate
*Beclomethasone
*Spiramycin
*Budesonide

#33
!A 50-year-old man is concerned about an increase in body temperature to 38C, a cough with hard-
to- separate green sputum. Objectively: breath rate - 24 per min. In the lungs, scattered buzzing
rales are heard, decreasing after coughing. In the analysis of sputum – 85% of neutrophils.
Which of the following drugs is most appropriate?
*Ampicillin
*Paracetamol
*+ Rovamycin
*Diflucan
*Biseptol
#34
!A 35-year-old man with community-acquired pneumonia, while taking amoxicillin 3.0 g / day, has
an increase in body temperature up to 380 ° C, increased cough with mucopurulent sputum.
What further antibacterial tactics are MOST appropriate?
*Add gentamicin
*Increase the dose of amoxicillin
*Replace amoxicillin with extensillin
*+ Replace amoxicillin with azithromycin
*Replace amoxicillin with amoxicillin / clavulanate

*!A 25-year-old male complains of weakness, a dry cough for 2 days, pain in the sternum, and a
fever up to 37.0 ° C, which appeared after hypothermia. History: frequent SARS (ARVI). Does not
smoke. By percussion is determined pulmonary sound. By auscultation is listened hard breathing,
no wheezing. Diagnosed - Acute Bronchitis.
Which of the following is the MOST probable pathogenetic mechanism?
*Reduction of the respiratory surface of the lungs
*Spasm of the smooth muscles of the respiratory tract
*Obstruction of the bronchi with a viscous secret
*+ Adhesion of pathogens on the walls of the bronchi
*Increased amount of mucus

#2
*!A 29-year-old female complains of a feeling of soreness and burning behind the sternum, a dry
cough, and weakness. History: frequent SARS (ARVI). On examination, pallor of the skin is noted.
Percussion is determined the pulmonary sound. By auscultation is listened hard breathing, no
wheezing.
Which of the following diagnoses is MOST probable?
*Spontaneous pneumothorax
*+ Acute bronchitis
*Lung cancer
*Aspiration pneumonia
*Dry pleurisy

#3
*!A student, 14 years old, after an acute respiratory illness suffered a dry, excruciating cough, pain
behind the sternum, shortness of breath. During auscultation in the lungs hard breathing, single dry
rales are heard. In the blood test: white blood cells 7.0 x 10 9 / l, ESR 18 mm / hour.
Which of the following diagnoses is MOST probable?
*+ Acute bronchitis
*Community-acquired pneumonia
*COPD
*Bronchial asthma
*Lung cancer

#4
*!A 25-years-old male complains of weakness, a dry cough for 2 days, pain behind the sternum,
and a fever up to 37.0 ° C, which appeared after hypothermia. History: frequent SARS (ARVI).
Does not smoke. Percussion determined a clear pulmonary sound. By auscultation is listened hard
breathing.
Which of the following diagnoses is MOST probable?
*Spontaneous pneumothorax
*Exudative pleurisy
*Lung cancer
*Community-acquired pneumonia
* +Acute bronchitis
#5
*!A 29-years-old female complains of a feeling of sore throat, a dry cough, pain behind the ster-
num, and weakness. History: frequent SARS (ARVI). On examination, the skin is of normal col-
or. Percussion determined pulmonary sound. By auscultation is listened hard breathing.
Which of the following indicators of the general blood test is MOST likely to increase?
*Red blood cells
*Platelets
*Eosinophils
*Neutrophils
*+ White blood cells

#6
*!A 29-years-old female complains of a feeling of rawness and burning behind the sternum, a dry,
painful cough, pain behind the sternum, weakness. History: frequent SARS (ARVI). On ex-
amination, pallor of the skin is noted. Percussion determined pulmonary sound. By auscultation
listened hard breathing, scattered dry wheezing. Preliminary diagnosis: Acute bronchitis.
Which of the following is the MOST probable pathogenetic mechanism?
*Immunological mechanisms
*Violation of gas exchange
*Irritation of the mucous membrane by chemicals
* +Violation of respiratory tract protection mechanisms
*Increased vascular permeability

#7
*!A 48-years-old male complains of a cough with mucopurulent sputum for a long time, an in-
crease of body temperature up to 37.8 ° C, weakness, malaise that appeared after hypothermia.
Objectively: cyanosis of the nasolabial triangle, thickening of the nails. Auscultation determined
hard breathing.
Which of the following examination methods is MOST justified for diagnosis?
*Echocardiography
*+X-ray
*Electrocardiography
*Peak flowmetry
*Pulse oximetry

#8
*!A 40-years-old female complains of a cough with mucopurulent sputum for a long time, an
increase of body temperature up to 37.0 0С, weakness, and malaise. From the anamnesis he
smokes for 12 years. Objectively: pallor of the skin. By auscultation is listened hard breathing.
Which of the following indicators of the general blood test is MOST likely to increase?
*Red blood cells
*Platelets
*Eosinophils
*Neutrophils
*+ White blood cells

#9
*!A 38-years-old female complains of coughing with mucopurulent sputum for a long time,
shortness of breath, weakness, and malaise. From the anamnesis smokes for 12 years. Objective-
ly: pallor of the skin. By auscultation is listened hard breathing, dry wheezing during forced ex-
piration.
Which of the following causes MOST often cause violations of obstructive pulmonary ventila-
tion?
* +Spasm of smooth muscles of the bronchi
*Accumulation of fluid in the pleural cavity
*Development of inflammatory infiltration in lung tissue
*Development of pneumotorax
*Deformation of the chest and spine

#10
*!A 48-years-old male complains of a cough with mucopurulent sputum for a long time, an in-
crease of body temperature up to 37.8 ° C, weakness, malaise that appeared after hypothermia.
Objectively: cyanosis of the nasolabial triangle, thickening of the nails. By auscultation listened
hard breathing. Preliminary diagnosis: Chronic bronchitis, exacerbation
According to the clinical protocol "Chronic bronchitis", Ministry of Health of the Republic of
Kazakhstan 2013, which of the main research methods is the most appropriate?
* General blood test
*Blood chemistry
* General urine analysis
* Sputum microscopy
* +General sputum analysis

#11
*!A 40-years-old female complains of a cough with mucopurulent sputum for a long time, an
increase of body temperature up to 37.0 0С, weakness, and malaise. From the anamnesis smokes
for 12 years. Objectively: pallor of the skin. By auscultation listened hard breathing with moist
rales, that disappear after coughing. Diagnosed with Chronic bronchitis, exacerbation.
Which of the following is the MOST characteristic in the pathogenesis of chronic bronchitis?
*Aspiration of the contents of the oropharynx
* +Development of hyperkrinia, dyskrinia, mucostasis
*Allergic reaction of the bronchial mucosa
*Violation of respiratory tract protection mechanisms
*Immunological mechanisms

#12
*!A 68-years-old female went to the doctor with complaints of cough with sputum, shortness of
breath, chest pain associated with breathing, fever, sweating, weakness. Sick after hypothermia.
Objectively: on the lower right lungs a shortened percussion sound. During auscultation, small
bubbling rales are heard there.
Which of the following diagnoses is MOST probable?
* +Community-acquired pneumonia of the lower lobe of the right lung
*Chronic obstructive bronchitis
*Bronchial asthma
*Chronic obstructive pulmonary disease
*Aspiration pneumonia

#13
*!A 27-years-old female was hospitalized with complaints of coughing, fever, shortness of breath,
and weakness. In the anamnesis after hypothermia (bathed in a river in the mountains) there was a
dry nasal cough, fever in the body, profuse sweating, weakness. On the 3rd day of
illness, “rusty” sputum began to separate. Which of the following types of pneumonia is MOST
likely?
* +Community-acquired
*Nosocomial
*Aspiration
*Immunodeficiency
*Interstitial

#14
*!A 52-years-old male complained of shortness of breath, cough with sputum, weakness, and low-
grade body temperature. A blood test for HIV infection is positive. During auscultation in all
departments of both lungs moist finely bubbling rales are heard. Which of the following diag-
noses is MOST probable?
*Community-acquired pneumonia
*Exudative pleurisy
* +Pneumonia in immunocompromised
*Aspiration pneumonia
*Bronchial asthma

#15
*!A 47 years old female is in intensive care unit after cholecystectomy. In the evening, her body
temperature rose to 38 °, pains appeared in the right half of her chest associated with breathing,
coughing. During auscultation on the right side below the angle of the scapula, listened moist,
finely bubbling rales, breath rate is 24 per minute. In the analysis of blood leukocytes 14 x 10 9 / l,
neutrophils - 8%, ESR 30 mm / h, CRP (+++). On the X-ray is an extensive infiltrative shadow of
the lower lobe of the right lung.
Which of the following diagnoses is MOST probable?
*Community-acquired right side lower lobe pneumonia
*Aspiration pneumonia
*Immunodeficiency pneumonia
* + Nosocomial right side lower lobe pneumonia
* Lung abscess

#16
*!A 65-years-old male turned to the therapist with complaints of cough with mucopurulent spu-
tum, fever up to 38 ° C, shortness of breath.
Which of the following descriptions MOST corresponds to the presented chest radiograph?
* +Inflammatory infiltration in the lower lobe of the right lung
*Destruction cavity in the lower lobe of the right lung
*Focal formation in the lower lobe of the right lung
*Bilateral dissemination
*Bilateral increase in lung roots

#17
*!A 24-years-old male complained of a cough with mucopurulent sputum, fever up to 38 ° C,
shortness of breath. Abuses alcohol and smoking. Objectively, there are traces of injections on the
hands, the smell of vomit.
Which of the following diagnoses is MOST characteristic for this chest X-ray?

* Fluid-filled sac pleurisy of the upper lobe of the right lung


* +Aspiration pneumonia of the upper lobe of the right lung
*Lung cancer
*Abscess of the upper lobe of the right lung
*Nosocomial pneumonia of the upper lobe of the right lung

#18
*!A 40-years-old female on an outpatient appointment with a district doctor of the clinic com-
plains of an increasing of body temperature up to 37.6 ° C in the evenings, persistent cough with
mucopurulent sputum, shortness of breath during physical exertion, general weakness, excessive
sweating. With auscultation in the lower parts of the right lung, weakened vesicular breathing is
listened. Which of the following examination methods is MOST justified for diagnosis?
*Spirography
* +X-ray
*Electrocardiography
*Peak flowmetry
*Pulse oxemitria

#19
*!A 30 years-old female, after severe hypothermia notes the appearance of pain in the right half of
the chest associated with breathing, fever up to 39 °, chills, nasal cough, sweating. Objective- ly:
facial flushing, Herpes labialis, breath rate - 26 per minute. In the lungs shortening of the per-
cussion sound on the right side below the angle of the scapula, here wet small-rales are heard. On
the X-ray, infiltration in the lower lobe of the right lung.
Which of the following diagnoses is MOST probable?
* +Community-acquired pneumonia of the lower lobe of the right lung
*Aspiration pneumonia of the lower lobe of the right lung
*Immunodeficiency pneumonia
*Nosocomial pneumonia of the lower lobe of the right lung
*Chronic bronchitis, exacerbation

#20
*!A 57-years-old male is in the intensive care unit for acute myocardial infarction. On the 3rd day
of hospitalization, body temperature increased to 38 ° C, a cough appeared with mucopuru- lent
sputum, sweating. During auscultation, small bubbling rales are heard in the lower right.
Which of the following diagnoses is MOST probable?
*Heart failure
*Aspiration pneumonia
*Right-sided pleurisy
* +Nosocomial pneumonia
*Dressler's syndrome

#21
*!A 24-years-old male complained of a cough with mucopurulent sputum, fever up to 38 ° C,
shortness of breath. Abuses alcohol and smoking. Objectively, there are traces of injections on the
hands, the smell of vomit. By auscultation in the lower parts of the right lung is listened weakened
vesicular breathing.
Which of the following diagnoses is MOST probable?
*Community-acquired pneumonia
* +Aspiration pneumonia
*Chronic bronchitis, exacerbation
*Exudative pleurisy on the right
*Lung cancer

#22
A 40-years-old male at an outpatient appointment with a district doctor of the clinic complained of
a temperature increase of up to 38 ° C in the evenings, persistent cough with mucopurulent sputum,
shortness of breath during physical exertion, general weakness, excessive sweating. He considers
himself sick during the week when the cough intensified, shortness of breath appeared during
walking, fever in the afternoon. By auscultation in the lower parts of the right lung is heard
weakened vesicular breathing.
Which of the following diagnoses is MOST probable?
* +Community-acquired right side lower lobe pneumonia
*Aspiration pneumonia
*Pneumonia in individuals with immunodeficiency
* Nosocomial right-side lower lobe pneumonia
* Chronic bronchitis, exacerbation
* #23
A 68-years-old female with complaints of cough with sputum, shortness of breath, chest pain
associated with breathing, fever, sweating, weakness. Sick after hypothermia.Which of the
following diagnoses is MOST characteristic for this chest X-ray?

* +Community-acquired lower-lobe pneumonia


*Chronic bronchitis
*Bronchial asthma
*Chronic obstructive pulmonary disease
*Lung cancer

#24
*!A 34-years-old male after an acute respiratory illness noted the appearance of cough with spu-
tum, chest pain, shortness of breath, fever up to 39.4 ° C. Objectively: a shortened percussion
sound in the lower lobe of the right lung. During auscultation, small bubbling rales are heard there.
Which of the following diagnoses is MOST probable?
*+Community-acquired pneumonia of the lower lobe of the right lung
*Chronic bronchitis
*Bronchial asthma
*Chronic obstructive pulmonary disease
*Exudative right-sided pleurisy

#25
*!A 58-years-old female turned to the therapist with complaints of cough with sputum, fever up to
38 0C, shortness of breath, severe sweating, weakness. From history, she took cytostatics and
hormones for rheumatoid arthritis for a long time. During auscultation, finely bubbling rales are
heard below the angle of the scapula on the right side.
Which of the following diagnoses is MOST probable?
*Community-acquired pneumonia
*Aspiration pneumonia
* +Pneumonia in immunocompromised persons
*Nosocomial pneumonia
*Exudative pleurisy

#26

A 75-years-old male is in intensive care unit after surgery on his stomach. In the evening, his body
temperature rose to 38 °, pains appeared in the right half of his chest, aggravated by cough- ing.
During auscultation on the right side below the angle of the scapula, is listened wet small- bubbling
rales, breath rate is 26 per minute.
Which of the following diagnoses is MOST probable?
*Community-acquired pneumonia
*Exudative pleurisy on the right
*Immunodeficiency pneumonia
* +Nosocomial pneumonia
*Dressler syndrome
#27
*!A 58-years-old female with complaints of cough with sputum, shortness of breath, chest pain
associated with breathing, increased body temperature, sweating, weakness. Sick after hypo-
thermia. During auscultation in the lower parts of the right lung, moist small-bubbling rales are
heard. Which of the following examination methods is most justified for a diagnosis?
*Spirography
* +X-ray
*Electrocardiography
*Peak flowmetry
*Pulse oxemitria

#28
*!A 45-years-old female with complaints of cough with mucopurulent sputum, fever up to 39 ° C,
shortness of breath.
Which of the following diagnoses is MOST characteristic for this chest X-ray?

* +Pneumonia of the lower lobe of the right lung


*Lung cancer
*COPD
*Bronchial asthma
*Pleurisy of the lower lobe of the right lung

#29
*!Sputum analysis: amount - 50 ml, color - rusty, nature - mucopurulent, consistency - viscous,
microscopy: epithelium - 1-2 in the field of view, leukocytes - 20-25 in the field of view, red blood
cells - 10-15 in the field of view , pneumococci ++, atypical cells - no, mycobacterium tu-
berculosis - no.
Which of the following diagnoses is MOST probable by sputum analysis?
*Acute bronchitis
* +Pneumonia
*Bronchial asthma
*Pleurisy
*Lung cancer

#30
*!A 54-years-old male after an acute respiratory illness appeared with cough with sputum, pain
behind the sternum, shortness of breath, fever up to 39.4 ° C. Objectively: breath rate is 26 per
minute. During auscultation in the lower parts of the right lung, moist small-bubbling rales are
heard. In the analysis of blood leukocytes 20x10 9 / l, the neutrophils - 8%, ESR 30 mm / h, CRP
(+++). The chest X-ray shows an extensive infiltrative shadow in the lower lobe of the right lung.
What severity of the disease is the most relevant patient’s data?
*Light
*Average
* +Heavy
*Moderate
*Very heavy

#31
*!A 25-years-old male with a diagnosis of Community-acquired pneumonia of the lower lobe of
the right lung received outpatient treatment. On the 7th day there are no complaints, the condi- tion
is satisfactory, by auscultation - vesicular breathing, no wheezing.
What further management tactics of the patient is MOST appropriate?
*Continue antibiotic therapy
*Prescribe another series of antibiotics
*Proceed to work
* +Control X-ray of chest on the 10th day
*Prescribe inhaled corticosteroids

#32
*!A 38-years-old female went to the doctor with complaints of cough with sputum, shortness of
breath, chest pain associated with breathing, fever, sweating, weakness. Sick after hypothermia.
Objectively: on the lower part of right lungs a shortened percussion sound. During auscultation,
small bubbling rales are heard there.
Which of the following diagnoses is MOST probable?
* +Community-acquired pneumonia of the lower lobe of the right lung
*Chronic obstructive bronchitis
*Bronchial asthma
*Chronic obstructive pulmonary disease
*Aspiration pneumonia

#33
*!A 22 years-old female complaints of asthma attacks, paroxysmal cough with difficult to sepa-
rate sputum, shortness of breath with little physical exertion, nasal congestion. Suffers from hay
fever from the age of 18. Objectively: a state of moderate severity. By percussion of the lungs - a
box sound. By auscultation, a large number of dry wheezing on the exhale is heard.
Which of the following diagnoses is MOST probable?
*Community-acquired lower-lobe pneumonia
*Chronic obstructive bronchitis
*Nosocomial upper lobar pneumonia
*Chronic obstructive pulmonary disease
* +Bronchial asthma, persistent course

#34
*!A 22 years-old female complaints of asthma attacks, paroxysmal cough with difficult to sepa-
rate sputum, shortness of breath during the small physical exertion, nasal congestion. Suffers from
hay fever from the age of 18. Objectively: a state of moderate severity. By percussion of the lungs -
a box sound. By auscultation, a large number of dry wheezing on the exhale is heard. Which of the
following examination methods is MOST justified for diagnosis?
* +Spirography with bronchodilator test
*Fibrobronchoscopy with biopsy
*Electrocardiography, echocardiography
*Computed tomography of the chest
*X-ray of the chest in two projections

#35
*!A 55-years-old male is diagnosed with bronchial asthma. Peak expiratory flow <60% of nor-
mal; daily lability >30%.
What severity of the disease is MOST consistent with the patient’s data in accordance with the
clinical protocol for the diagnosis and treatment of bronchial asthma (MH RK, 2019)?
*Intermittent
*Persistent light
*Persistent Medium
* +Persistent heavy
*Very heavy
#36
*!A 34-years-old female, after home cleaning appeared shortness of breath, coughing attacks with
mucous viscous sputum, wheezing. The patient has a history of allergy to honey, herbs, dust.
In a blood test, an increasing of which indicator is the MOST expected?
*Red blood cells
*White blood cells
*Platelet count
*Lymphocytes
* +Eosinophils

#37
*!A 39-years-old female with complaints of coughing at night, nasal congestion, sneezing, lac-
rimation. Marks allergy to wormwood bloom. Spirometry data: VC - 88%, FVC - 65%, FEV1 -
68%, FEV1 / VC - 79%.
What is the violation of the function of external respiration MOST consistent with spirometry data?
*Restrictive mild
*Restrictive expressed
*Mixed
* +Obstructive mild
*Obstructive moderate

#38
*!A 40-years-old female is diagnosed with bronchial asthma. Peak expiratory flow ≥ 80% of the
norm, daily lability of 20% - 30%.
What severity of the disease is MOST consistent with the patient’s data in accordance with the
clinical protocol for the diagnosis and treatment of bronchial asthma (MH RK, 2019)?
*Intermittent
* +Persistent light
*Persistent Medium
*Persistent heavy
*Very heavy

#39
*!A 22 years-old female complaints of asthma attacks, paroxysmal cough with difficult to sepa-
rate sputum, shortness of breath during small physical exertion, nasal congestion. Suffers from hay
fever from the age of 18. Preliminary diagnosis: Bronchial asthma.
Which of the following laboratory examination methods is MOST justified for diagnosis?
*Sputum analysis
* +IFA for IgE
*Definition of CRP
*IFA for IgM
*Blood on CYFRA

#40
*!A 22 years-old female complaints of asthma attacks, paroxysmal cough with difficult to sepa-
rate sputum, shortness of breath during small physical exertion, nasal congestion. Suffers from hay
fever from the age of 18. Diagnosed with bronchial asthma, persistent course.
For daily self-monitoring of their condition, which of the following is MOST appropriate?
*Tonometer
* +Peak flow meter
*Thermometer
*Pulse Oximeter
*Blood glucose meter

#41
*!A 34-years-old female after home cleaning, appeared shortness of breath, coughing attacks with
mucous viscous sputum, wheezing. The patient has a history of allergy to honey, herbs, dust. In the
general analysis of sputum - eosinophils 7%, the level of IgE is increased on the im- munogram.
Spirography data: VC - 85%, FEV1 - 50%, Tiffno index - 65%.
Which of the following diagnoses is MOST probable?
*Chronical bronchitis
*Community-acquired pneumonia
*Hospital pneumonia
*Bronchiectatic disease
* +Bronchial asthma

#42
*!A 39-years-old female with complaints of coughing at night, nasal congestion, sneezing, lac-
rimation. Marks allergy to wormwood bloom.
In sputum analysis, which of the following indicators is MOST likely to increase?
*Epithelium
* +Eosinophils
*White blood cells
*Red blood cells
*Pneumococci

#43
*!A 45-years-old male is diagnosed with bronchial asthma. Peak expiratory flow ≥ 80% of nor-
mal, daily lability less than 20%.
What severity of the disease is MOST consistent with the patient’s data in accordance with the
clinical protocol for the diagnosis and treatment of bronchial asthma (MH RK, 2019)?
* +Intermittent
*Persistent light
*Persistent medium
*Persistent heavy
*Very heavy

#44
*!A 55-years-old female is diagnosed with bronchial asthma. Peak expiratory flow 60-80% of the
norm, daily lability> 30%.
What severity of the disease is MOST consistent with the patient’s data in accordance with the
clinical protocol for the diagnosis and treatment of bronchial asthma (MH RK, 2019)?
*Intermittent
*Persistent light
* +Persistent medium
*Persistent heavy
*Very heavy

#45
*!A 52-years-old male complained of cough with sputum that was difficult to separate, shortness of
breath at rest, intensifying during physical exertion. From the anamnesis: the last 10 years
notes a constant cough in the morning and sputum production. Smokes for 30 years Objectively: a
barrel-shaped chest, by percussion - pulmonary sound with a box-like shade. During ausculta- tion,
breathing is weakened, exhalation is elongated, dry scattered rales are heard.
Which of the following examination methods is MOST justified for diagnosis?
*Bronchoscopy
*Peak flowmetry
*Pulse oximetry
* +Chest x-ray
*Spirometry

#46
*!A 45-years-old male with a diagnosis of Chronic obstructive pulmonary disease. On spirome-
try, - VC - 85%, FEV1 - 60%, Tiffno index - 68%.
What severity is the most consistent with spirometry data?
*Easy
* +Average
*Moderate
*Heavy
*Extremely severe

#47
*!A 62-years-old male, a driver, smokes 40 years. Complaints of shortness of breath, cough with
mucous sputum during the day. Objectively: by auscultation of the lungs - weakened breathing, dry
wheezing in all fields.
Which of the following diagnoses is MOST probable?
* +COPD
*Bronchitis
*Pneumonia
*Idiopathic pulmonary fibrosis
*Pleurisy

#48
*!A 62-years-old male, a driver, smokes 40 years. Complaints of shortness of breath, cough with
mucous sputum during the day. Objectively: by auscultation of the lungs - weakened breathing, dry
wheezing in all fields. Preliminary diagnosis: Chronic obstructive pulmonary disease. Spi- rometry
was performed.
Which of the following spirometry indicators is MOST likely to decrease?
*Lung capacity
*MOS25
*Maximum ventilation
*Tidal volume
* +Tiffno Index

#49
*!A 69-years-old male with complaints of shortness of breath at the slightest exertion. During
spirometry, FVC parameters were determined - 39%, FEV1 - 23%, FEV1 / FVC - 66%. What type
of respiratory dysfunction is the most relevant these data?
* +Very severe obstructive
*Severe restrictive
*Obstructive mild
*Obstructive severe
*Mixed type
#50
*!In chronic obstructive pulmonary disease, which of the following complications is MOST like-
ly?
*Nephritis
* +Chronic pulmonary heart
*Lung abscess
*Metapneumonic pleurisy
*Asthmatic status

#51
*!A 52-years-old male complained of cough with sputum that was difficult to separate, shortness of
breath at rest, intensifying during physical exertion. From the anamnesis: the last 10 years notes a
constant cough in the morning and sputum production. Smokes for 30 years. Objectively: a barrel-
shaped chest, by percussion pulmonary sound with a box-like shade. During ausculta- tion,
breathing is weakened, exhalation is elongated, dry scattered rales are heard.
Which of the following diagnoses is MOST probable?
*Community-acquired pneumonia
*Chronical bronchitis
*Pleurisy
*Bronchial asthma
* +Chronic obstructive pulmonary disease

#52
*!A 52-years-old male with a diagnosis of Chronic obstructive pulmonary disease. When con-
ducting spirometry, VC - 65%, FEV1 - 45%, Tiffno index -55%.
What severity is the most consistent with spirometry data?
*Easy
*Secondary
*Moderate
* +Heavy
*Extremely severe

#53
*!A 75-years-old male with a diagnosis of Chronic obstructive pulmonary disease. When con-
ducting spirometry, VC - 65%, FEV1 - 25%, Tiffno index -28%.
What severity is the most consistent with spirometry data?
*Easy
*Secondary
*Moderate
*Heavy
* +Extremely heavy

#54
*!A 62 years0old male, a driver, smokes 40 years. Complaints of shortness of breath, cough with
mucous sputum during the day. Objectively: with auscultation of the lungs, weakened breathing,
dry wheezing in all fields. Preliminary diagnosis: Chronic obstructive pulmonary disease.
Which of the following diagnostic methods will be the MOST informative?
*Peak flowmetry
* +Spirometry
*Bronchoscopy
*Fluoroscopy
*Electrocardiography

#55
*!A 62 years-old male, a driver, smokes for about 40 years. Complaints of shortness of breath,
cough with mucous sputum during the day. Objectively: the barrel-shaped chest, percussion
pulmonary sound with a boxed shade. During auscultation, breathing is weakened, exhalation is
elongated, dry scattered rales are heard. Diagnosed Chronic Obstructive Pulmonary Disease.
Which of the following changes in chest X-ray is the MOST characteristic for this patient?
*Homogeneous intense dimming with oblique upper boundary
*Intense round infiltrative dimming
*Rounded cavity with a smooth inner contour
*Focal-infiltrative changes with fuzzy, uneven contours
* +Increased airiness of lung tissue, low standing diaphragm

#56
*!A decrease in which of the following spirometric indicators is MOST typical for patients with
COPD according to GOLD (2018)?
* +FEV1 and FEV1 / FVC
*FVC and MOS 25%
*MOS 50% and MOS 75%
*VC and FVC
*FVC and MVL

*!MOST expected duration of the acute bronchitis


*Week 1
* +2 weeks
*1 month
*2 months
*3 months

#2
*!Which of the following is the MOST probable cause of acute bronchitis?
* +Viral infection
*Nosocomial infection
*Fungal infection
*Bacterial infection
*Oropharynx’s pathogenic flora of the (with aspiration)

#3
*!MOST likely risk factor for acute bronchitis
*Hormonal disorders
*Malnutrition
*Sedentary lifestyle
* +Violation of nasal breathing
*Obesity

#4
*!Which of the following is the MOST characteristic in the pathogenesis of acute bronchitis?
*Development of obstructive syndrome
*Violation of mucociliary transport
*Hyperreactivity of the bronchi
* +Violation of the mechanisms of natural defense
*Immunological disorders

#5
*!The MOST specific clinical sign of chronic bronchitis is
* +Cough for 3 months a year for 2 consecutive years
*Cough worse at night, asthma attacks
*Chest pain associated with breathing, frequent and shallow breathing
*Hemoptysis, sweating, weight loss, low-grade fever
*Stunning chills, fever, severe intoxication symptoms

#6
*!Which of the following complications of chronic bronchitis is the MOST frequent?
*Pneumothorax
* +Pneumonia
*Lung gangrene
*Lung cancer
*Dry pleurisy

#7
*!Which of the following is the MOST probable cause of the community-acquired pneumonia?
*Bronchoscopy
*Postoperative period
*Alcoholism
*Diabetes
* +Hypothermia

#8
*!Which of the following is the MOST probable cause of aspiration pneumonia?
*Hormone therapy
*Postoperative period
* +Alcoholism
*Diabetes
*Hypothermia

#9
*!The MOST common causative agent of pneumonia is ...
* +Pneumococcus
*Mycobacterium tuberculosis
*Cytomegalovirus
*Anaerobic staphylococci
*E. coli

#10
*!A 30 years old male. Complaints about shortness of breath, cough with white viscous sputum. He
works as an operator at a poultry farm. Objectively: height - 170 cm, weight - 80 kg, BMI -
27.7 kg \ m2. The skin is of normal color. By auscultation of the lungs, dry wheezing is heard,
more in the upper and middle sections.
Which of the following is the MOST likely cause of shortness of breath?
*Age and gender
*Stress
* +Profession
*Smoking
*Obesity

#11
*!A 30 years old male. Complaints about shortness of breath, cough with white viscous sputum.
Works as an operator on television. Objectively: height - 170 cm, weight - 110 kg, BMI - 38 kg \
m2. The skin is of normal color. By auscultation of the lungs, dry wheezing is heard more in the
upper and middle sections. Preliminary diagnosis: Bronchial asthma.
Which of the following is the MOST characteristic in the pathogenesis of the disease?
*Emphysema
* +Bronchospasm
*Laryngospasm
*Pneumosclerosis
*Mucostasis

#12
*!Which of the following is the MOST characteristic pathogenetic mechanism in bronchial
asthma?
* +Bronchial hyperreactivity
*The presence of exudate in the alveoli
*Inflammatory reaction of the lungs to gas
*Pulmonary hypertension
*Destruction of the alveolar walls

#13
*!A 30 years old male. Complaints about shortness of breath, cough with white viscous sputum.
Works as an operator on television. Objectively: height - 170 cm, weight - 110 kg, BMI - 38 kg \
m2. The skin is of normal color. By auscultation of the lungs, dry wheezing is heard more in the
upper and middle sections. Preliminary diagnosis: Bronchial asthma.
Which of the following is the MOST characteristic in the pathogenesis of the disease?
*Emphysema
* +Bronchospasm
*Laryngospasm
*Pneumosclerosis
*Mucostasis

#14
*!A 60-year-old woman with a diagnosis of Chronic obstructive pulmonary disease. She worked as
a cook for many years.
Which of the following is the MOST likely cause of the disease?
*Α1-antitrypsin deficiency
*Bad habits
*Age and gender of the patient
* +Occupational hazard
*Duration of lung disease #15
*!A 58 years old female, a doctor. Complains of cough with mucous sputum, shortness of breath
during physical exertion. Suffers from chronic obstructive pulmonary disease for 10 years. Does
not smoke. Sister revealed pulmonary emphysema.
Which of the following is the MOST likely cause of the disease?
*Age and gender of the patient
*Occupational hazards
*Air pollution
*Duration of the disease
* +An α1- antitrypsin insufficiency

#16
*!THE MOST probable research method for detecting a violation of bronchial obstruction in acute
bronchitis
* + Spirometry
*X-ray
*Bronchoscopy
*Pulse oximetry
*Electrocardiography

#17
*!Which of the following diagnoses is MOST characteristic for the infiltrative changes with fuzzy
contours on X-ray?
* +Pneumonia
*Chronical bronchitis
*Bronchial asthma
*Lung cancer
*Bronchiectatic disease

#18
*!The MOST characteristic changes in the lungs on X-ray examination of a patient with
pneumonia is?
* +Focal and infiltrative changes
*Destruction cavities
*Increased airiness of lung tissue
*Thickening of the walls of the bronchi
*Basal pneumosclerosis

19.
*!A 30 years old male is diagnosed with bronchial asthma. Spirometry with bronchodilator test was
performed.
By how many percent is the MOST expected increase in FEV1 with the positive test?
*2
*4
*5
*8
* +12

#20
*!What are the most likely changes in skin color for respiratory failure?
*"Cold" cyanosis
* + "Warm" cyanosis
*Acrocyanosis
*Pallor of the skin
*"Blush" on the cheeks

#21
*!The MOST specific sign of respiratory failure is:
*General weakness
*Chest pain
* + Dyspnea
*Swelling on the face
*Headaches

#22
*!For respiratory failure of the III degree the MOST characteristic is:
*Shortness of breath with significant physical exertion
*Shortness of breath at the slightest exertion
* +Dyspnea at rest
*Bradycardia during exercise
*Tachycardia with significant physical exertion

Endocrinology 1уровень

#23
*!Which of the following is the most common sign of diabetes mellitus?
* Hypocoagulation.
* Hyperglycemia.
* Hyperuricemia.
* Hypernatremia
* Hyperkalemia.

#24
*!Which of the following are risk factors for the development of type 2 diabetes mellitus ?
* Frequent exposure to viral infections.
* The presence of bad habits in the history.
* Lack of weight.
* Poor diet.
++*Genetic predisposition.

#25
*!At what level of fasting venous plasma glucose likely diagnosis of diabetes?
+*Glucose level equal to or above 6.0 mmol / l.
* Glucose level equal to or above 8.0 mmol / l.
* Glucose level equal to or above 9.0 mmol / l.
* Fasting glucose level equal to or above 7.0 mmol/l.
* Fasting glucose level equal to or above *0 mmol/l.
#26
*!What can you think about if the patient has fasting glycemia in the venous blood 7 mmol/l, two
- hours after meals – 1*5 mmol / l?
+*About diabetes mellitus.
* About impaired fasting glycemia.
* About impaired glucose tolerance.
* On the absence of carbohydrate metabolism disorders.
* About necessity of carrying out the glucose tolerance test.

#27
*!The patient is 60 years old with type 2 diabetes mellitus, taking hypoglycemic drugs for 10
years. The level of HbA1c is 1*2%. What state of carbohydrate metabolism corresponds to the
level of glycosylated hemoglobin in this patient and further tactics of the doctor?
* Increasing the dose of drugs that increase insulin secretion.
* Increasing the dose of drugs that increase the sensitivity of tissues to insulin.
* Review the patient's diet and continue the previous treatment.
*+Add to the treatment of long-acting insulin in the evening.
* Add to the treatment of rapid-acting insulin before meals.

#28
*!What is the main pathogenesis of diffuse toxic goiter?
* The formation of antibodies to trophophase.
* Formation of antibodies to thyroglobulin.
++* Formation of antibodies to TSH receptors.
* The formation of antibodies to thyroid peroxidise.
* Formation of antibodies to ATP-AZE.

#29
*!Female 35 years receiving treatment with a diagnosis of diffuse toxic goiter II degree,
thyrotoxicosis of moderate severity, takes Thyrozol and Anaprilin. What parameters it is
necessary to evaluate the effectiveness of the treatment of this patient?
*The level of blood pressure.
++* By heart rate.
* Dynamics of weight recovery.
* The size of the thyroid gland.
* Regression of eye symptoms.

#30
*!In a 59-year-old patient with a diagnosis of diffuse toxic goiter II degree with severe
thyrotoxicosis, conservative treatment for *5 years did not have an effect, there are
contraindications to surgical treatment. Your further medical tactics?
* Increase the dose of thyrostatics.
++*Refer to radioiodtherapy.
* To add to the treatment of glucocorticoids.
* Add physiotherapy techniques.
* Continue treatment with thyrostatics for a long time.

#31
*!What corresponds to the morphological picture of autoimmune thyroiditis?
* Purulent-inflammatory process
+++*Lymphoid infiltration.
* Cystic degeneration.
* Atypia follicle.
* Connective tissue proliferation.

#32
*!What is caused by primary hypothyroidism?
* Diseases of the pituitary gland.
* Insensitivity of thyroxine in the peripheral tissues.
* Diseases of the hypothalamus.
* +++Thyroid disease.
* A decrease in the activity of thyroxine.

#33
*!Patient 40 years, about persistent constipation. Due to the dryness of the skin, low blood
PRESSURE and a rare heart rate during the examination is aimed at examining the functions of
the thyroid gland. TRH-8.2 IU/l, free T4 – 7.1 pmol / l. how is the patient's thyroid condition
assessed?
* Euthyroidism.
* Hyperthyroidism.
* Subclinical hypothyroidism.
* Subclinical hyperthyroidism.
* +++Manifest hypothyroidism.

#34
*!How should I take thyroxine drugs?
*+++Once daily on an empty stomach, 30-40 minutes before meals.
* Three times a day after meals.
* Once daily in the evening after meals.
* Once daily before bedtime.
* Through day once a day before meals at any time.

#35
*!What division of endocrine is affected in the Conn's syndrome ?
* Adrenal medulla.
* +++The zonaglomerulosa of the adrenal gland.
*The zone reticularis of the adrenal glands.
* The zona fasciculate of the adrenal glands.
* The anterior pituitary gland.

#36
*!What is produced by excessive amounts of pheochromocytoma hormones?
*+++ Catecholamines.
* Cortisol.
* Insulin.
* Endorphins.

#37
*!A 39-year-old woman presents with complaints of frequent headaches, weight gain by 24
kg for the last 2 years. On physical examination the face is moon-shaped, excessive hair growth
on the face, especially fat accumulate in the upper body, purple striae on the front surface of the
abdomen. The heart rate is regular at 88 beats/min. Blood pressure is 170/100 mmHg. On CT
bilateral hyperplasia of adrenal, on MRI of the pituitary-adenoma size 1, 1x1,2x1, 2cm. What is
the most likely diagnosis in this patient?
* Conn’s syndrome.
*++ Cushing's disease.
* Pheochromocytoma.
* Graves ' disease.
* Adison's disease.

#38
*!What is the likely diagnosis for polydipsia and polyuria, combined with a low specific gravity
of urine?
* Diabetes mellitus.
* Diabetes insipidus.
* Psychogenic polydipsia.
* Chronic renal failure.
* Urinary tract infection.
2 уровень
#57
*!What hormone level increase is expected in the presence of obesity, galactorrhea,
dysmenorrhea in the patient?
* ACTH.
* STG.
* FSH.
* TSH.
*+++ Prolactine PRL.

#58
*!What is characterized by primary chronic insufficiency of the adrenal cortex?
* Local depigmentation of the skin.
* ++Diffuse hyperpigmentation of the skin and mucous membranes.
* Hypertensive crisis.
* Constantly high blood pressure.
* Weight gain.

#59
*!In a 27-year-old woman suffering from chronic adrenal insufficiency, acute deterioration with
high temperature, nausea, vomiting, signs of dehydration, hemorrhagic eruption, drop in blood
pressure, hyperkalemia, hyponatremia was noted against the background of stress. What is your
diagnosis?
* Hemorrhagic vasculitis.
* Acute intestinal infections.
* +++Acute adrenal insufficiency.
* Diabetes insipidus.
* sympathoadrenal crisis.

#60
*!WhatisC-Peptide?
* Hormone
* Enzyme
* Neurotransmitter
* Vitamin
*+++ Part of the insulin molecule

#61
*! What causes in diabetes mellitus development of polydipsia and polyuria?
* joining inflammatory diseases of the urinary tract
* psycho-emotional state
*++++ increased osmotic pressure of blood and urine
* over-stimulation of the thirst centre
* hyperkaliemia

#62
*!What is the level of blood glucose in 2 hours after eating allows you to diagnose diabetes
mellitus?
+++* blood glucose level equal to or above 10.0 mmol/l
* blood glucose level equal to or above 1*0 mmol/l
* blood glucose level equal to or above 1*0 mmol/l
* blood glucose level equal to or above 9.0 mmol/l
* blood glucose level equal to or above 1*0 mmol/l

#63
*! If a 45-year-old patient in the venous blood fasting glucose level of 10 mmol/l, 2 hours after
eating – 14 mmol / l, titer of antibodies to GAD increased, what is the likely diagnosis?
* diabetes mellitus type 2
* violation of fasting glucose
* violation of glucose tolerance
* there are no carbohydrate metabolism disorders
++++* diabetes mellitus type 1

#64
*!In 58-year-old patient during preoperative preparation revealed an increase in the level of
glucose in the venous blood on an empty stomach-6.8 mmol/ l, with repeated determination of
fasting glucose-6.1 mmol/l, 2 hours after eating – 10.8 mmol / l. What is the diagnosis of the
patient?
* violation of glucose tolerance
* violation of fasting glucose
* +++diabetes mellitus type 2
* steroid diabetes
* diabetes mellitus type 1

#65
*!The patient is 20 years old, type 1 diabetes was diagnosed 5 years ago, receives insulin therapy
according to the intensified scheme. The level of glycosylated hemoglobin was 8.8%. What is
the state of carbohydrate metabolism corresponds to the level of Nva1s in this patient and further
tactics of the doctor?
*Decompensation, increasing the dose of short-acting insulin 
*Decompensation, increasing the dose of insulin of prolonged action 
*Compensation, continue treatment with the same dose of insulin 
*Decompensation, the dose of insulin to double due to the expiration of 5 years 
+++*The decompensation correction dose of insulin for glycemic profile

#66
*!What is most likely associated with the development of diffuse toxic goiter?
* Inflammatory process in the thyroid tissue 
*Growth of connective tissue in the thyroid gland 
*Congenital defect of the thyroid gland 
*Radiation exposure of the thyroid gland 
*++++A defect of the immunological system

#67
*!A 49-year-old patient receiving treatment with a diagnosis of diffuse toxic goiter, during the
next visit to the doctor complains of darkening of the skin and increased weakness. When
viewed, diffuse hyperpigmentation of the skin and a decrease in diastolic pressure. The patient,
perhaps, the development of a complication of the disease?
* thyrotoxic heart 
* encephalopathy 
* ophthalmopathy 
* diabetes 
++++* adrenal insufficiency

#68
*!Patient 25 years old, diagnosed with diffuse toxic goiter II degree, severe thyrotoxicosis, takes
tyrosol and anaprilin for 6 months. What is the maximum duration of conservative treatment in
this patient?
* 3-6 months 
* 1 year 
* 5 years 
++++* 1-1, 5 years 
* 3 years

#69
*!What manifests Hoshimoto’s thyroiditis?
* purulentthyroidits
* subacute thyroiditis
* tuberculosis of the thyroid gland
++* autoimmune thyroiditis
* postpartum thyroiditis

#70
*!Accumulation of which substances in the body characteristic of hypothyroidism?
* acidicaminoglycosides
* amyloid’s
* collagen
* lipoproteins
* water and sodium

#71
*!The patient is 35 years old, from 14 years old takes daily 100 mcg of L-thyroxine. No history
of surgery, no radiation. TSH level is *0 ME/L. ultrasound of the thyroid gland – atrophy of the
gland. What could be the cause of hypothyroidism in this patient?

* postoperative hypothyroidism
* autoimmune thyroiditis
* Postradiation hypothyroidism
* +++congenital aplasia of the thyroid gland
* endemic goiter

#72
*!After what time will determine the level of TSH to control the patient who takes thyroxine?
* everymonthчерезкаждыймесяц
*++1 time in 2-3 months
*1 time in 6 month
* 1 time per year
* 1 time per week

#73
*!What part of the endocrine system is affected by pheochromocytoma?
* anterior pituitary gland 
* beam area of the adrenal glands 
* the mesh area of the adrenal glands 
* glomerular area of the adrenal glands 
* +++adrenal medulla

#74
*!What is characteristic of syndromes Icenko-Kushinga?
*losing weight
* fever
*++++ uneven obesity
* dryskin
* hypotonia

#75
*!A 40-year-old woman complains of headaches, weight gain over the past *5-2 years, menstrual
irregularity. When viewed, pay attention to the moon-shaped face, excessive hair growth on the
face, thinning of the skin, uneven obesity with predominant deposition on the upper shoulder
girdle, in the region of the 7 cervical vertebra, the presence of crimson color of the striae on the
skin of the thighs and abdomen. The level of cortisol in the blood-850 nmol / l. What is your
further tactics of examination?
*+++CT of the adrenal glands, MRI of pituitary, holding the sample dexamethasone 
* Ultrasound of the adrenal glands, EEG, testing with insulin 
*Ultrasound of the pelvic organs, pituitary MRI, the sample with clonidine 
* Ultrasound of the abdominal cavity, glucose tolerance test 
* CT of adrenal glands, ECG, testing with adrenaline
#76
*!The test with 1-24АКТГ (synaktenom) is carried out with the aim of:

 ACTH deficiency detection


 Diagnosis of primary hyperaldosteronism
 +++Diagnosis of primary hypocorticism
 Differential diagnosis of the disease and Cushing's syndrome
 Preoperative preparation of patients with secondary hypocorticism

#77
*!Specify what the relative density of urine is characteristic of diabetes insipidus?
+++* 1000-1005
* 1010-1015
* 1015-1020
* 1020-1025
* 1025-1030

#78
*!What group of drugs is used to treat prolactinoma?
*antidiuretic hormone 
* antidepressants 
*+++ cabergolines 
* sulfonylurea derivatives 
* aldosterone antagonists

#79
*!What is characterized by secondary adrenal insufficiency?
* low aldosterone levels while maintaining the concentration of cortisol in the blood 
* a low concentration of cortisol with a high ACTH level 
+++* a reduced concentration of cortisol and ACTH 
* persistent increase in blood pressure 
* hyperpigmentation of the skin

#80
*!The patient has acute adrenal insufficiency. Which of the following measures would be
correct?
++++* hydrocortisone intravenously jet and drip, infusion of 0.9% sodium chloride and
5% 
glucose solution 
* prednisolone inside, infusion of 5% glucose solution 
* hydrocortisone intramuscularly, infusion of 0.9% sodium chloride solution 
* hydrocortisone intravenously struino, dopamine 
* hydrocortisone intravenously jet, inside mineralocorticoid

#81
*! Glycated hemoglobin is indicator of …?
* Fat metabolism
* Hemolytic disorder
++* Carbohydrate metabolism
* Blood coagulation system
* protein metabolism

#82
*!What is the main thing in the pathogenesis of type 1diabetes mellitus?
* excessive secretion of glucagon by pancreatic alpha cells
* excessive secretion of somatostatin by delta pancreatic cells
++++* absolute lack of insulin secretion by pancreatic beta cells
* insufficient secretion of glucagon by pancreatic alpha cells
* absolute deficiency of secretion of somatostatin by delta cells of the pancreas

#83
*!What level of glycated hemoglobin confirms the diagnosis of diabetes?
* 6,0%
* 7,0%
* 5,0%
++* 6,5%
* 7,5%

#84
*! A 30-years-old patient,suffers from bronchial asthma and receives dexamethasone. It was
found an increase level of glycemia to 1*5 mmol / l. What is the probable diagnosis of the
patient?
* impaired glucose tolerance
* impaired fasting glucose
* type 2 diabetesmellitus
+* steroid diabetes
* type 1diabetesmellitus

#85
*!A 55-years-old patient during the annual survey was revealed an increase level of glycemia in
venous blood to 6.5 mmol / l, whereupon level of glycemia in venous blood was determined
again and its level was 5,9mmol/l. What should doctor do?
*todiagnose diabetes and prescribe treatment
++* to measurement glucose tolerance test
* noneedsanyexaminations
* remeasurementfasting blood sugar level in venous blood
* remeasurementfasting blood sugar levelin capillary blood after 3 months

#86
*!A 26-year-old patient with a diagnosis of type 1 diabetes mellitus needs a conclusion by an
endocrinologist before elective surgery. Level ofglycatedhemoglobinis 6,5%. What state of
carbohydrate metabolism corresponds to the course of diabetes in a given patient and is he
admitted to surgical treatment?
* decompensation, not allowed for surgical treatment until compensation is achieved
* decompensation, allowed for surgical treatment right now
*+++ compensation, allowed for surgical treatment
* compensation, not allowed forsurgical treatment
* decompensation, allowed for surgical treatment until compensation is achieved

#87
*!Which kind of effect have antibodies to TSH receptors on the body?
* increased iodine absorption
* increaserate of thyrocytes
* increase production of thyroid stimulating hormone
++* stimulation of thyroid hormone production
* increase volume of thyrocyte

#88
*!A 35-year-old man with a diffuse toxic goiter suddenly deteriorated after stress: heartbeat,
shortness of breath, trembling of the whole body, severe weakness, sweating, nausea, vomiting,
and loose stools. The patient's speech is slurred, the skin is hot, moist, excited, the temperature is
39.0 ° C, the heart rate is 170 in 1 minute, the heart sounds are loud, the rhythm is broken, the
blood pressure is 150/40 mm Hg. What complication had the patient developed?
* acute heart failure
* pheochromocytoma
* hypertensive crisis
* hypoglycemic condition
++++* thyrotoxic crisis

#89
*!If a patient with a diffuse toxic goiter has hyperpigmentation of the skin attached to the main
symptoms and there is a decrease in diastolic pressure, what examination should be performed to
rule out adrenal insufficiency?
* Metanephrine in urine
* aldosterone in the blood
* testosterone in the blood
+++* cortisol in the blood
* estrogen in the blood

#90
*!What is a frequent complication of autoimmune thyroiditis?
* thyrotoxic crisis
* malignancy
* anemia
* ++ hypothyroidism
* displacement of the organs of the neck and mediastinum

#91
*!What hormonal change is characteristic for hypothyroidism?
* excess TSH and thyroid hormones
+++ * thyroid hormone deficiency
* excess thyroid hormones, suppression of TSH production
* excessive secretion of thyroid hormones at normal levels of TSH
* deficiencyofTSHandthyroidhormones

#92
*!A 59-year-old patient due to the presence of nodes in the thyroid gland 2 months ago
underwent an operation of thyreoidectomy and was prescribed replacement therapy with L-
thyroxine at a dose of 100 µg. Which hormone should be controlled further to assess the thyroid
state of this patient?
* free T4
* TTG, free T4
*++ TSH
* Antibodies to receptor TTG
* antibodies to thyreoglobulin

#93
*!What is the duration of thyroxin in the diagnosis of congenital hypothyroidism in a child?
* until reaching adulthood
* until growth is complete
* 1-2 years
++* for whole life
* before menopausal age

#94
*!What is the definition of Cushing syndrome?
* hyper aldosteronism
* estroma
* ++glucoseteroma
* insuloma
* pheochromocytoma
3 уровень
#35
*!What is characteristic for Conn syndrome?
* hypoglycemia
* hyponatremia
++* hypokalemia
* hypomagnesemia
* hyperkalemia

#36
*!A 24-year-old patient has complaints of headaches, absence of menstruation, weight gain,
excessive growth of hair on the face and along the white line of the abdomen. The above
complaints appeared during the year. Objectively: height - 168 cm, weight - 92 kg, purple stretch
marks with a “minus-tissue” on the skin of the chest, abdomen, thigh. AP - 150/100 mm Hg.
MRI of the pituitary gland is an adenoma 0.8 cm in diameter.
Thelevelofcortisolinthebloodishigh. Whatisyourpreliminarydiagnosis?
++* Cushing disease
* Conn's syndrome
* arterial hypertension
* Cushing syndrome
* metabolic syndrome

#37

*!What examination is necessary to a man with complaints of headaches, blurred vision, weight
gain, decreased libido, increased level of prolactin in the blood?
++* MRI of the brain
* Breast ultrasound
* CT of the adrenal glands
* Thyroid scintigraphy
* Ultrasound of the abdominal organs

#38

*!Which of the following drugs is prescribed in the treatment of diabetes insipidus?


++++* minirin
* aspirin
* furosemide
* maninil
* metformin

#39
*!A 35-year-old woman is diagnosed with primary chronic adrenal insufficiency of moderate
severity. What treatment will be correct?
* glucocorticoid oral pills
*++ glucocorticoids in combination with mineralocorticoids oral pills
* Mineralocorticoids oral pills
* glucocorticoid injections daily
* glucocorticoid intravenous drip in saline

#40
*!Which of the following reasons can cause acute adrenal insufficiency?
* ++septicemia, DIC - syndrome, massive hemorrhage in the adrenal cortex
* tuberculosis, cancer, lung actinomycosis
* Cushing disease
* systemicconnectivetissuediseases
* syphilis

#41
*!Addiction of patients with hypocorticism to salty food is MOST likely caused by:
 ++sodium loss
 dehydration
 hyperthermia
 convulsions
 diarrhea

#42
*! Diffuse hyperpigmentation of the skin is characteristic for:
 Cushing disease
 ++Addisson Disease
 Babinsky-Fröhlich disease
 Hypothyroidism
 Acromegaly

#43
*! The most informative laboratory method of diagnosis of pheochromocytoma is
  determination of one of the following substances:
 renin
 cortisol
 ++metanephrine
 aldosterone
 corticosterone

#44
*!The morphological substrate of the Cushing syndrome is:
 thymoma
 ++corticosteroma
 corticotropinoma
 aldosteroma
 androsteroma

#45
*! One of the listed symptoms is a differential diagnostic criterion of primary and secondary adrenal
insufficiency:
 Weakness
 Loss of weight
 Abdominal pain
 Loss of appetite
 ++Hyperpigmentation of the skin

#46
*!The MOST advisable therapy of acute adrenal insufficiency:
 antibacterial therapy
 anticonvulsant therapy
 vitamin therapy
 ++hydrocortisone replacement therapy
 anti-inflammatory drug therapy

#47
*!* Chronic adrenal insufficiency manifests against the backdrop of one of the following factors:
 carbohydrate food abuse
 decrease of temperature with hypothermia
 elderly or old age
 pronounced menopausal syndrome
 ++decompensation of comorbidities

#48
*!The place of catecholamine synthesis:
 Anterior pituitary gland
 Zona reticularis
 ++Adrenal medulla
 Zona fasciculata
 The glomerular zone of the adrenal glands

#49
*!What’s the main mechanism of the osteoporosis’ pathogenesis in Cushing syndrome?
 lipolysis
 ++Catabolism
 Anabolism
 Gluconeogenesis
 Glycogenolysis

#50
*!A 42-year-old woman with a long-term destabilization of BP has general weakness, dizziness,
and frequent urination at night. Objectively BP-200/110. On electrocardiography: normal sinus
rhythm. The electrical axis of the heart has horizontal position. Ps-75 per minute. High teeth R-
V5, V6, Deep teeth S-V1, V* Biochemical analysis: total cholesterol-6.5 mmol / l,
threeglycerides-*1 mmol / l, Na-144 mmol / l, K-*8 mmol / l. The level of aldosterone (when
lying) was elevated, renin-reduced activity.
What is the most likely cause of increased blood pressure?
 adrenal insufficiency
 nephrotic syndrome
 ++primary hyperaldosteronism
 essential hypertension
 vegetate-vascular dystonia, hypertonic type

#51
*!A 28-year-old woman complains of marked weakness, fatigue, enhanced pigmentation of
exposed skin, increased need for salty foods. 4 years ago, she suffered focal pulmonary
tuberculosis, was registered at a dispensary in a TB dispensary. Low power, skin
hyperpigmented, there are dark spots on the mucous membrane of the gums and in places of
friction, muffled heart sounds. Pulse-78 in 1 min. HELL -90 / 60 mm Hg In the blood: eritr-3,2
mln., Leuke-4,0 thousand, Nv-85 g / l, ESR-12 mm / h, blood sugar at 8.00- *9 mmol / l.

What diagnosis is MOST likely?

 orthostatic hypotension
 pituitary insufficiency
 anemic syndrome
 ++adrenal insufficiency
 hypoglycemic syndrome

#52
*!Female 47 years old. Complaints of bouts of increased blood pressure, accompanied by
redness, fear, palpitations, lasting from several minutes to several hours. In the blood: HB -143 g
/ l, Erythrocytes - *5 * 1012 / l, leukocyte - 1*0 * 109 / l, ESR - 16 mm / h. Blood sugar 6.2
mmol / l. In urine, the specific gravity is 1015, protein-avs, glucose - +. The content of
metanephrine and free catecholamines in the blood increased by 6 times.

Prescription of the drug from which group is MOST advisable?

* Diuretics

* ACE inhibitors

* ++Alpha blockers

* Calcium channel blockers


* Angiotensin receptor blockers

#53
*!A woman was delivered to the emergency room with an increase in blood pressure up to
300/130 mm Hg. The crisis was accompanied by anxiety, tachycardia, polydipsia. Blood tests
showed an increase in glucose up to 20 mmol / l.

What drug is MOST justified for the relief of these symptoms?

 calcium channel antagonists


 mineralocorticoid antagonists
 nootropics
 ++selective alpha adrenergic blocker
 desensitizing agents

#54
*!A 42-year-old woman suffering from long-term destabilization of blood pressure notes general
weakness, dizziness, and frequent urination at night. When viewed from HELL 200/110
mm.rt.st. In the urine: specific gravity 1009. BHA: total cholesterol-6.5 mmol / l, triglycerides-
*1 mmol / l, LDL-4 mmol / l, HDL-*8 mmol / l. Na-144 mmol / l, K-*8 mmol / l. On ECG: sinus
rhythm, correct. The horizontal position of the EOS. HR-75 per minute. High teeth R-V5, V6,
Deep teeth S-V1, V*

The MOST reliable screening method in diagnosis is?

 blood electrolytes (Na, K, Cl)


 determination of cortisol levels
 ++determination of the level of aldosterone
 small dexamethasone test
 aldosterone-renin ratio

#55
*!Clinical manifestations of androsteroma due to the increase:

 secretion of aldosterone and estrogen


 secretion of glucocorticoids and aldosterone
 adrenaline and norepinephrine production
 estrogen production
 ++androgen production

#56
*!The test with 1-24АКТГ (synaktenom) is carried out with the aim of:
 ACTH deficiency detection
 Diagnosis of primary hyperaldosteronism
 ++Diagnosis of primary hypocorticism
 Differential diagnosis of the disease and Cushing's syndrome
 Preoperative preparation of patients with secondary hypocorticism

# 91

 *! The patient has been observed in the clinic for 15 years due to elevated sugar levels. Against
the background of an acute respiratory disease, the smell of acetone from the mouth appeared;
according to the keto test, the urine reacted positively to acetone.
 What is the MOST probable threatened condition that can develop with the further progression
of this condition?

 ++* ketoacidosis
 *dehydration
 * hypoglycemia
 * acetonemic vomiting
 * respiratory failure

# 93

 *! A 67-year-old man has fasting sugar of 9.5 mmol / l after eating-14.8 mmol / l. The glycated
hemoglobin is at 10% level. The patient refuses the prescribed insulin therapy due to the need
to make daily injections.
 Which of the following complications of insulin therapy is MOST likely?
 * anisocytosis
 * hypokalemia
 ++* lipodystrophy
 * thrombocytopenia
 * fatty hepatosis

# 94

 *! A girl of 17 years old came with complaints of an increase in neck size, irritability, tearfulness,
heart palpitations, weight loss. Hyperhidrosis is noted, red dermographism is expressed. Found
2 degree enlargement of the thyroid gland, exophthalmos.
 What mode of the day is recommended to her the MOST?
 *free
 ++*bed rest
 * half-bed rest
 * strict bed rest
 * classes in the group of exercise therapy

35
 *! A 38-year-old man complains of fatigue and dizziness in the last 6 months. He lost 8 kg in 3
months, complains of periodic abdominal pain. The blood pressure is 100/50 mm RT. Art.,
generalized hyperpigmentation. The cousin has type 1 diabetes.
 What treatment is MOST indicated?
 * nootropics
 * prokinetics
 +* glucocorticodes
 * vegetotropic drugs
 * desensitizing therapy

* « Nephrology block »* 1 * 7 *

*! True bacteriuria is the excretion of conditionally pathogenic flora in the amount of urine (microbial
bodies in 1 ml of urine):

*1000

*5.000

*10.000

+*100.000

*1000.000

#2

*! The MOST common causative agent of community-acquired urinary tract infection is:

+*E.coli

*Proteus

*Citrobacter

*Ps. aerouginosa

*Staphylococcus

#3

*! Specify the MOST probable diagnosis with a combination of proteinuria, edema, hypertension and
hematuria:

*cystitis
* pyelonephritis

+* glomerular pathology

* interstitial jade

* urinary tract infection

#4

*! Indicate the cause of the development of ascites characteristic of nephrotic syndrome:

*+ loss of protein in the urine


* allergic reaction
* decreased protein synthesis
* endocrine disorders
* metabolic disorders

#5
*! In shock, the cause of acute kidney damage is:
* vegetative disorders
* concomitant infection
* formation of complexes AG-AT
* +drop in blood pressure
* the effect of toxic substances from damaged tissues

#6

*! Indicate the mechanism of development of chronic kidney disease with a decrease in the number of
functioning nephrons:

*++ hyperfiltration

* obstructive uropathy

* vasculitis of the renal vessels

* immune glomerular damage

* microbial inflammatory damage to the kidneys

#7
*! What is the role of hyperfiltration for chronic kidney disease?
* improves kidney function
* does not affect kidney function
*+universal progression mechanism
* has a sclerosing effect only with pyelonephritis
* has a sclerosing effect only with glomerulonephritis
* «Nephrology» block * 2 * 19 *

#8

*! Patient M., 25 years old. Received with complaints of dull pain in the lumbar region, more to the
right, fever up to 39 ° C, with chills, excretion of turbid urine. Objectively: the skin is of a normal color,
there is no edema, the right kidney is palpated, painful, blood pressure 100/70 mm Hg In the blood test:
white blood cells 16 x 109 / l, ESR 40 mm / h. In the analysis of urine: beats. weight 1009-1016, protein
0.66 g / l, in the urinary sediment, leukocytes are completely, red blood cells are 1-2 in p / s, hyaline
cylinders are single. Which of the following preliminary diagnoses is MOST probable?

* nephroptosis

+* acute pyelonephritis

*urolithiasis disease

* acute glomerulonephritis

*chronic pyelonephritis

#9

*! Patient M., 25 years old. Received with complaints of dull pain in the lumbar region, more to the
right, fever up to 39 ° C, with chills, excretion of turbid urine. Objectively: the skin is of a normal color,
there is no edema, the right kidney is palpated, painful, blood pressure 100/70 mm Hg In the blood test:
white blood cells 16 x 109 / l, ESR 40 mm / h. In the analysis of urine: beats. weight 1009-1016, protein
0.66 g / l, in the urinary sediment, leukocytes are completely, red blood cells are 1-2 in p / s, hyaline
cylinders are single. Which of the following research methods is the MOST important?

* kidney biopsy

* cystography, ultrasound of the kidneys

* cystoscopy, ultrasound of the kidneys

+* tank. urine culture, kidney ultrasound

* excretory urography standing

#10

*! Indicate which of the following with asymptomatic bacteriuria DOES NOT need antibiotic therapy?

+* teenager
*pregnant
* patient with diabetes
* patient with atony of the bladder
* patient after kidney transplantation

#11

*! The choice of an antibacterial agent for the treatment of urinary tract infection depends on:

* degrees of bacteriuria

* degrees of leukocyturia

+* microorganism sensitivity

* the presence of urinary tract obstruction

* lack of urinary tract obstruction

#12

*! Indicate the MOST common cause of urinary tract infection in women:

* the presence of obstructive uropathy

*+ short urethra and anus proximity

* kidney abnormalities are more common

* bladder dysfunction

* the presence of vesicoureteral reflux

#13

*! What morphological picture corresponds to rapidly progressive glomerulonephritis?

+* extracapillary

* minimal changes

* membranous nephropathy

* mesangioproliferative

* membrane proliferative

#14
*! Acute glomerulonephritis with nephritic syndrome should be treated with:

* monotherapy with prednisone

* combination of prednisone and heparin

* detoxification therapy, plasma

* combination of prednisolone with cytostatic

+* salt-free diet, symptomatic therapy

#15

*! The man is 32 years old. On examination: proteinuria, recurrent macrohematuria. Morphologically


established: diffuse proliferation of mesangial cells and IgA deposits in the mesangium. What course is
characteristic of this disease?

* acute cyclic

* always favorable

* favorable in men

* fast progressive

* ++chronic with gradual sclerosis of the kidneys

???#16

*! A 21-year-old man has swelling on his legs, blood pressure is normal. Blood creatinine is 80 μmol / l,
total protein is 40 g / l, albumin is 15 g / l and proteinuria is 6 g / s, urinary sediment is normal. What
research is needed to clarify the diagnosis?

* Amburge test

*++ kidney biopsy

* Zimnitsky test

* Nechiporenko test

*tank. urine culture with antibiotic determination

??#17

*! The main symptom characteristic of rapidly progressive glomerulonephritis?

* hematuria
* proteinuria

* severe intoxication

* high blood pressure

*+ rapid increase in blood creatinine

#18

*! What is the main pathogenetic mechanism of prerenal and renal AKI?

* DIC

* metabolic acidosis

* antiphospholipid syndrome

* violation of proximal reabsorption

* ++decreased filtration due to renal hypoperfusion

#19

*! Indicate why hyperkalemia is dangerous?


* pulmonary edema
* itchy skin
* respiratory arrest
*++ cardiac arrest
* massive peripheral edema

#20

*! As a result of what medications does AKI develop more often?

* macrolides

* penicillins

* co-trimaxazole

* cephalosporins

* ++aminoglycosides

#21

*! For the period of recovery of diuresis of acute renal failure is characteristic:


* oliguria
* polyuria

* isostenuria

* normal urinalysis

* urine specific gravity

#22

*! The action of angiotensin-converting enzyme inhibitors:

* constriction of efferent arterioles

*+++ arterioles extension

* arteriole enlargement

* increased synthesis of angiotensin 2

* angiotensin 2 receptor inhibition

#23

*! A 40 year old patient has type 1 diabetes. Examination of urine revealed a protein of 0.165 g / l. What
is the preliminary diagnosis and treatment tactics?

* diabetes mellitus, pyelonephritis, antibiotics

* diabetes mellitus, glomerulonephritis, prednisone

* diabetes mellitus, acute renal failure, transfer to the dialysis department

* diabetes mellitus, chronic renal failure, transfer to the dialysis department

*++ type 1 diabetes mellitus, diabetic nephropathy, nephroprotection

#24

*! The patient is 33 years old. Observed for type I diabetes mellitus for 15 years. In a control study in
urine tests: proteinuria 0.066 g / l, urinary sediment without pathologyGlomerular filtration rate = 56
ml / min. Which drug is the MOST preferred in this situation?

* diuretics

* cytostatics

* antibiotics

*++ ACE inhibitors


* glucocorticosteroids

??#25

*! 56 year old patient with arterial hypertension. With a routine examination: blood creatinine 140 ml /
min. GFR = 56 ml / min. In OAM: protein 0.165 g / l, urinary sediment is normal. Which of the
preliminary diagnoses is MOST probable?

* acute glomerulonephritis, nephritic syndrome

* hypertension, acute renal failure

* arterial hypertension, chronic kidney disease, stage 4

*++++ hypertensive nephropathy, chronic kidney disease stage 3

* chronic glomerulonephritis, chronic kidney disease stage 3

#26

*! Which of the treatment methods is radical in chronic renal failure?

*hemodialysis

* plasmapheresis

* hemofiltration

+* kidney transplant

* peritoneal dialysis

* Nephrology block * 3 * 12 *

#27

*! What definition corresponds to the term “empirical antibiotic therapy” of a urinary tract infection?

* one-time treatment of urinary tract infection

* prescribing treatment taking into account antibiogram

* treatment of urinary tract infections with nitrofurans

* treatment of urinary tract infections with sulfonamides

*++ prescription of broad-spectrum antibiotics without results of sowing


#28

*! What you need to remember about aminoglycosides when prescribed to patients with kidney
disease:

* have a wide spectrum of action

* act on gram-positive flora

+++ have a nephrotoxic effect

* have an advantage in the treatment of amyloidosis

* have an advantage in the treatment of glomerulonephritis

#29

*! What disease is indicated for the administration of fluoroquinolone antibiotics:

* systemic disease

* urinary tract infections in children

*++ urinary tract infections in adults

* post-infectious acute glomerulonephritis

* postinfectious interstitial nephritis

#30

*! What clinical syndrome is MOST characteristic for acute post-streptococcal glomerulonephritis?

* nephrotic syndrome

* ++acute nephritic syndrome

* isolated urinary syndrome

* chronic nephritic syndrome

* rapidly progressive glomerulonephritis

#31

*! What is true about a kidney biopsy?

* not accompanied by complications


* contraindicated in persons with a kidney transplant

* allows you to choose an adequate treatment protocol

* shown in all patients with nephrotic syndrome

* contraindicated in nephrotic syndrome in adults

#32

*! The patient is 32 years old. Marks a rash on the body, "butterfly on the face", arthralgia, fever.
Nonsteroidal drugs are prescribed. Swelling appeared throughout the body. In the urine - protein 5
g / day, red blood cells 10-15 in s / sp, cylinders 2-3 in s / sp. Which of the following preliminary
diagnoses is MOST probable?
* renal artery thrombosis

* nephritis associated with zoonotic infection

* acute glomerulonephritis, nephritic syndrome

*++ nephritis associated with systemic lupus erythematosus

* acute tubulointerstitial nephritis associated with NSAIDs

# 33

*! Which of the following conditions poses a direct threat to life and requires quick correction in a
patient with AKI?

* hyperuricemia

++* hyperkalemia

* hyperphosphatemia

* increase in blood urea

* increased creatinine in the blood

# 34

*! What threatens the patient in the stage of decreased urine output in acute renal failure?

++* pulmonary edema

*dehydration

* hypokalemia

* arterial hypotension
* respiratory failure

# 35

*! The main measures in the development of AKI to maintain water-electrolyte and energy balance in
the body:

* quick transfer to parenteral nutrition

* the introduction of bicarbonate and a solution of sodium, potassium

*++ sufficient introduction of physiological solutions and proteins

* providing calories with an even ratio of protein, fat and carbohydrates

*fluid restriction according to losses and providing caloric intake with protein restriction

# 36

*! What level of glomerular filtration rate corresponds to CKD-3 st., Which even in the absence of
proteinuria requires the appointment of nephroprotective therapy:

* 90-120 ml / min

* 89-60 ml / min

* 59-30 ml / min

* 29-15 ml / min

* below 15 ml / min

#37

*! The patient is 31 years old. In childhood, he suffered from recurrent IMS against a
background of congenital kidney anomaly. Operated in childhood. Currently in urine: protein 0.3
g / l, white blood cells 3-4 in p / sp, red blood cells -2-3 in p / sp. GFR - 45ml / min. HB-90 g / l.
Correction, what disorders are indicated at this stage of CKD?
* hormonal
* metabolic
* immunological
* blood coagulation
++* anemia and proteinuria

#38

*! The patient is 17 years old. Diagnosis: chronic glomerulonephritis. CKD, pre-dialysis stage established.
What is NOT right in the prescribed treatment:

* protein restriction diet, ACE inhibitors, statins


* treatment of anemia, protein restriction diet

* treatment of hypertension and phosphorus restriction

* ++cyclosporin A, a diet with an increase in protein

* treatment of osteodystrophy, with phosphorus restriction

Program control on the topic:


Acute bronchitis
I variant
1. The MOST expected duration of acute bronchitis:
1) 1 week
+2) 2 weeks
3) 1 month
4) 2 months
5) 3 months
2. Which of the following is THE MOST likely cause of acute bronchitis?
+1) Viral infection
2) Nosocomial infection
3) Fungal infection
4) Bacterial infection
5) Pathogenic flora of the oropharynx ( during aspiration)

3. A 25-year-old man complains of weakness, dry cough for 2 days, pain behind the sternum,
increased body temperature to 37.0 0C, which appeared after hypothermia. In anamnesis:
frequent acute respiratory viral infection. Do not smoke. Percussion is determined by the
pulmonary sound. Auscultatory auscultated breathing hard, no wheezing. Diagnosed with:
Acute bronchitis.
Which of the following is THE MOST likely pathogenetic mechanism?
1) Reduction of the respiratory surface of the lungs
2) spasm of the smooth muscles of the respiratory tract
3) Blockage of the bronchi with a viscous secret
4) adhesion of pathogens on the walls of the bronchi
+5) Increase the amount of mucus secreted

4. A 25-year-old man complains of weakness, dry cough for 2 days, pain behind the sternum,
increased body temperature to 37.2 0C, which appeared after hypothermia. In anamnesis:
frequent acute respiratory viral infection. Do not smoke. Percussion is determined by the
pulmonary sound. Auscultation is listened to hard breathing, single dry wheezes. Diagnosed
with: Acute bronchitis.
Which of the following indicators of the General blood test IS MOST likely to
increase?
1) Red blood cells
2) Platelets
3) Eosinophils
++4) White blood cells
5) Neutrophils

5. A 29-year-old woman complains of a feeling of stinging and burning behind the sternum, dry
cough, weakness. In anamnesis: frequent acute respiratory infections. On examination, the pallor
of the skin is noted. Percussion is determined by the pulmonary sound. Auscultatory auscultated
hard breathing, no wheezing.
Which of these diagnoses IS MOST likely?
1) Spontaneous pneumothorax
++2) Acute bronchitis
3) Lung Cancer
4) Aspiration pneumonia
5) Dry pleurisy

6. THE MOST LIKELY method of investigation for detecting violations of bronchial patency in
acute bronchitis:
1) Spirometry
++2) Radiography
3) Bronchoscopy
4) Pulse Oximetry
5) Electrocardiography

7. A 14-year-old student who had an acute respiratory illness developed a dry, painful cough,
chest pain, and shortness of breath. During auscultation in the lungs, hard breathing and
occasional dry wheezing are heard. In the blood test: white blood cells 7.0 x 10 9/l, ESR 18
mm/h. In the sputum analysis-exfoliated epithelium, single macrophages. Spirography data: VC
– 80%, FVC– 80%, FEV1 - 65%, IT-85%.
Which of these diagnoses IS MOST LIKELY?
+1) Acute bronchitis
2) Community-acquired pneumonia
3) COPD
4) Bronchial asthma
5) Lung Cancer

8. A 19-year-old student had a cough with mucopurulent discharge on day 3 after suffering from
an acute respiratory illness.
Which of the following methods of non-drug therapy is MOST AIMED at relieving sputum
discharge?
1) Vibration massage
2) Physical education
3) Sanation of foci of infection
+4) Physiotherapy
5) Hardening of the body

9. The student went to the clinic with complaints of a dry, painful cough, pain behind the
sternum. Auscultation dry rales. Independently took mukaltin for 5 days without effect.
The appointment of any of the following groups of drugs is MOST appropriate?
1) Anticholinergic drugs
2) Antileukotriene drugs
3) Mukolitiki
4) b-2 short-acting agonists
5) Antihistamines

10. A 19-year-old student diagnosed with Acute bronchitis received outpatient treatment for 7
days. There are no complaints at the repeated reception, the condition is satisfactory.
Auscultative vesicular respiration, no wheezing.
What is THE MOST APPROPRIATE management strategy for a patient with acute bronchitis?
1) Appointment of long-acting b-2 agonists
2) Prescribing other types of antibiotics
3) Nebulizer therapy with corticosteroid
++4) Spa treatment
5) The designation of b-2 agonists short-acting

Program control on the topic:


Acute bronchitis
II variant
1. THE MOST LIKELY risk factor for acute bronchitis
1) Hormonal disorders
2) Poor nutrition
3) Sedentary lifestyle
++4) Violation of nasal breathing
5) Obesity

2. Which of the following is THE MOST CHARACTERISTIC in the pathogenesis of acute


bronchitis?
1) Development of obstructive syndrome
2) Violation of mucociliary transport
3) Hyperreactivity of the bronchi
+4) Violation of natural defense mechanisms
5) Immunological disorders

3. A 25-year-old man complains of weakness, dry cough for 2 days, pain behind sternum,
increased body temperature to 37.0 0C, which appeared after hypothermia. In anamnesis:
frequent acute respiratory viral infection. Do not smoke. Determined by percussion a clear
pulmonary sound. Auscultatory auscultated breathing hard.
Which of these diagnoses IS MOST LIKELY?
1) Spontaneous pneumothorax
2) Exudative pleurisy
3) Lung Cancer
4) Community-acquired pneumonia
++5) Acute bronchitis

4. Woman 29 years complains of a feeling of rawness in the throat, dry cough, pain behind
sternum, weakness. In anamnesis: frequent acute respiratory infections. On examination, the skin
is normal colors. Percussion is determined by the pulmonary sound. Auscultative listening to
hard breathing, scattered dry wheezes on the exhalation.
Which of the following indicators of the General blood test is MOST LIKELY to increase?
1) Red blood cells
2) Platelets
3) Eosinophils
4) Neutrophils
++ 5) White blood cells

5. A 29-year-old woman complains of a feeling of stinging and burning behind the sternum, dry
painful cough, pain behind the sternum, weakness. In anamnesis: frequent acute respiratory
infections. On examination, the pallor of the skin is noted. Percussion is determined by the
pulmonary sound. Auscultation is heard hard breathing, scattered dry wheezing. Preliminary
diagnosis: Acute bronchitis.
Which of the following is THE MOST LIKELY pathogenetic mechanism?
1) Immunological mechanisms
2) Violation of gas exchange
3) Irritation of the mucous membrane with chemicals
+4) Violation of the protection mechanisms of the respiratory tract
5) Increased vascular permeability

6. A 29-year-old woman complains of a sore throat, cough with scanty mucosa of sputum, pain
behind the sternum, weakness. In anamnesis: frequent acute respiratory infections. On
examination, the pallor of the skin is noted. Determined by percussion a clear pulmonary sound.
Auscultatory auscultated breathing hard. Preliminary diagnosis: Acute bronchitis. According to
the clinical Protocol "Acute bronchitis", MH RK 2016. Which additional research method is
most appropriate?
1) General analysis of blood
+2) Bacteriological analysis of sputum
3) General urinalysis
4) Blood test for cytokines
5) General sputum analysis

7. A 19-year-old student had a dry, painful cough and chest pain after suffering from an acute
respiratory illness. During auscultation in the lungs, hard breathing and occasional dry wheezing
are heard. In the blood test: white blood cells 5.0 x 10 9/l, ESR 14 mm / h. He was diagnosed
with acute bronchitis.
Which of the following methods of non-drug therapy IS MOST APPROPRIATE?
1) Physical education
+2) Physiotherapy
3) Sanation of foci of infection
) Diet compliance
5) Hardening of the body

8. The woman of 30 years diagnosed with ARVI. Acute bronchitis, obstructive syndrome.
Which of the following medications IS MOST APPROPRIATE?
1) Azithromycin
2) Bromhexine
3) Ambroxol
4) Pulmicort
++5) Salbutamol

9. A 19-year-old student diagnosed with Acute bronchitis received outpatient treatment for 7
days. There are no complaints at the repeated reception, the condition is satisfactory.
Auscultative vesicular respiration, no wheezing.
What IS THE MOST APPROPRIATE management strategy for a patient with acute bronchitis?
1) Appointment of long-acting b-2 agonists
2) Prescribing other types of antibiotics
3) Nebulizer therapy with corticosteroid
4) Spa treatment
5) The designation of b-2 agonists short-acting

10. A 17-year-old student after suffering an acute respiratory illness appeared dry, painful cough,
chest pain, shortness of breath. During auscultation in the lungs, hard breathing and occasional
dry wheezing are heard. In the blood test: white blood cells 7,0 X10 1 /l, ESR - 18 mm/hour. In
sputum analysis, the exfoliated epithelium is isolated macrophages. Spirography data VC-80%,
FVC-80%, FEV1-65%, the Tiffno Index Is 85%.
Which of these diagnoses IS MOST LIKELY?
+1) Acute bronchitis
2) Community-Acquired pneumonia
3) COPD
4) Bronchial asthma
5) Lung Cancer

Program control on the topic:


Pneumonia
І variant

1.Which of the following is the MOST likely cause of development community-acquired pneumonia?
1) Bronchoscopy
2) Postoperative period
3) Alcoholism
4) Diabetes mellitus
+5) Hypothermia

2. A 68-year-old woman consulted a doctor with complaints of coughing up phlegm, shortness of breath,
pain in chest associated with breathing, fever, sweating, weakness. Got sick after hypothermia.
Objectively: on the right lower parts of the lungs, shortening percussion sound. On auscultation, fine
bubbling rales are heard in the same place.
Which of the following diagnoses is MOST likely?
+1) Community-acquired pneumonia of the lower lobe of the right lung
2) Chronic obstructive bronchitis
3) Bronchial asthma
4) Chronic obstructive pulmonary disease
5) Aspiration pneumonia

3. A 27-year-old woman was admitted to the hospital with complaints of cough, increased body
temperature, shortness of breath, weakness. In anamnesis after hypothermia (bathed in river in the
mountains) there was a dry hacking cough, fever in the body, profuse sweating, weakness. On the 3rd day
of illness, "rusty" sputum began to separate. Which of the following types of pneumonia MOST likely?
1) Community-acquired
2) Nosocomial
3) Aspiration
4) Immunodeficiency
5) Interstitial

4. A 52-year-old man complains of shortness of breath, cough with phlegm, weakness, subfebrile body
temperature. A blood test for HIV infection is positive. Auscultation in all parts of both lungs is
auscultated moist fine bubbly wheezing. Which of the following diagnoses is MOST likely?
1) Community-acquired pneumonia
2) Exudative pleurisy
3) Pneumonia in immunocompromised
4) Aspiration pneumonia
5) Bronchial asthma

5. A woman, 47 years old, is in intensive care after cholecystectomy. In the evening she the body
temperature rose to 38 °, pains appeared in the right half of the chest, associated with breathing, cough.
On auscultation to the right below the angle of the scapula in the patient humid fine bubbling rales are
heard, NPV 24 per minute. In a blood test leukocytes 14 х10 9/ l, stab neutrophils - 8%, ESR 30 mm /
hour, CRP (+++). On radiograph an extensive infiltrative shadow of the lower lobe of the right lung.
Which of the following diagnoses is MOST likely?
1) Community-acquired lower lobe pneumonia on the right
2) Aspiration pneumonia
3) Immunodeficiency pneumonia
4) Nosocomial lower lobe pneumonia on the right
5) Lung abscess

6. A 65-year-old man turned to a therapist with complaints of cough with mucopurulent sputum, fever up
to 38 ° C, shortness of breath.
Which of the following descriptions MOST matches the given chest x-ray?
1) Inflammatory infiltration in the lower lobe of the right lung
2) The cavity of destruction in the lower lobe of the right lung
3) Focal formation in the lower lobe of the right lung
4) Bilateral dissemination
5) Bilateral enlargement of the roots of the lungs

7. A 24-year-old man complains of a cough with mucopurulent sputum, increased body temperature up to
38 ˚С, shortness of breath. Abuse alcohol and smoking. Objectively, there are traces of injections on the
hands, the smell of vomit. Which of the following MOST typical for this chest x-ray?
1) Encapsulated pleurisy of the upper lobe of the right lung
2) Aspiration pneumonia of the upper lobe of the right lung
3) Lung cancer
4) Abscess of the upper lobe of the right lung
5) Nosocomial pneumonia of the upper lobe of the right lung

8. A 40-year-old woman at an outpatient appointment with a local doctor of a polyclinic complains an


increase in temperature up to 37.6 ° C in the evenings, persistent cough with mucopurulent sputum,
shortness of breath during exertion, general weakness, increased sweating. On auscultation in the lower
parts of the right lung, weakened vesicular respiration. Which of the following survey methods is the
MOST is it justified to make a diagnosis?
1) Spirography
2) Radiography
3) Electrocardiography
4) Peak flowmetry
5) Pulse oxemitria

9. A 45-year-old man was diagnosed with community-acquired pneumonia.


What drugs are the MOST appropriate to prescribe?
1) Bronchodilators
2) Vitamins
3) Antispasmodics
4) Antibiotics
5) Corticosteroids

10. A 30-year-old woman was diagnosed with community-acquired pneumonia of the lower lobe of the
right lung. Prescription of which drug of the listed MOST appropriate?
1) Salbutamol
2) Amoxiclav
3) Prednisolone
4) Seretid
5) Euphyllin
Program control on the topic:
Pneumonia
ІІ variant
1. Which of the following is the MOST likely cause of aspiration pneumonia?
1) Hormone therapy
2) Postoperative period
3) Alcoholism
4) Diabetes mellitus
5) Hypothermia

2. A 30-year-old woman after severe hypothermia notes the appearance of pain in the right half of the
chest, associated with breathing, an increase in body temperature up to 39 °C, chills, hacking cough,
sweating. Objectively: facial hyperemia, Herpes labialis, NPV - 26 per minute. In the lungs, the
shortening of the percussion sound is on the right below the angle of the scapula, here wet fine bubbling
rales are heard. On the radiograph, infiltration in lower lobe of the right lung.
Which of the following diagnoses is MOST likely?
1) Community-acquired pneumonia of the lower lobe of the right lung
2) Aspiration pneumonia of the lower lobe of the right lung
3) Immunodeficiency pneumonia
4) Nosocomial pneumonia of the lower lobe of the right lung
5) Chronic bronchitis, exacerbation

3. A 57-year-old man is in the intensive care unit for an acute heart attack myocardium. On the 3rd day of
hospitalization, the body temperature increased to 38 ° C, appeared cough with mucopurulent sputum,
sweating. On auscultation in the lower right departments, fine bubbling rales are heard.
Which of the following diagnoses is MOST likely?
1) Heart failure
2) Aspiration pneumonia
3) Right-sided pleurisy
4) Hospital-acquired pneumonia
5) Dressler's syndrome

4. A 24-year-old man complains of a cough with mucopurulent sputum, increased body temperature up to
38 ˚С, shortness of breath. Abuse alcohol and smoking. Objectively, there are traces of injections on the
hands, the smell of vomit. On auscultation in the lower parts of the right lung, weakened vesicular
breathing.
Which of the following diagnoses is MOST likely?
1) Community-acquired pneumonia
2) Aspiration pneumonia
3) Chronic bronchitis, exacerbation
4) Exudative pleurisy on the right
5) Lung cancer

5. A 40-year-old man at an outpatient appointment with the district doctor of the polyclinic complained
for an increase in temperature up to 38 ° C in the evenings, constant cough with mucopurulent
sputum, shortness of breath on exertion, general weakness, increased sweating. Considers himself sick
during the week, when the cough intensified, shortness of breath appeared with walking, temperature in
the afternoon. On auscultation in the lower regions the right lung weakened vesicular breathing.
Which of the following diagnoses is MOST likely?
1) Community-acquired lower lobe pneumonia on the right
2) Aspiration pneumonia
3) Pneumonia in persons with an immunodeficiency state
4) Nosocomial lower lobe pneumonia on the right
5) Chronic bronchitis, exacerbation
6. A 68-year-old woman with complaints of coughing up phlegm, shortness of breath, chest pains
associated with breathing, fever, sweating, weakness. I got sick after hypothermia.
Which of the following diagnoses is MOST typical for this X-ray organs of the chest?
1) Community-acquired lower lobe pneumonia
2) Chronic bronchitis
3) Bronchial asthma
4) Chronic obstructive pulmonary disease
5) Lung cancer

7. Which of the following diagnoses is MOST typical for radiological pictures in the form of infiltrative
changes with fuzzy contours?
1) Pneumonia
2) Chronic bronchitis
3) Bronchial asthma
4) Lung cancer
5) Bronchiectasis

8. In an X-ray examination of a patient with pneumonia, the MOST characteristic changes in the lungs
are?
1. Focal-infiltrative changes
2. Cavities of destruction
3. Increasing the airiness of the lung tissue
4. Thickening of the walls of the bronchi
5. Basal pneumosclerosis

9. On the third day you are seeing a 35-year-old patient diagnosed with community-acquired pneumonia.
Is he receives antibacterial therapy (penicillin). Despite this the state of health does not improve, the body
temperature has not returned to normal.
What is the MOST advisable tactic of patient management?
1) Change to cephalosporin antibiotics
2) Increase the dose of antibiotic you receive
3) Add bronchodilators to treatment
4) Continue treatment with the selected antibiotic
5) Prescribe inhaled corticosteroids

10. A 23-year-old woman was diagnosed with community-acquired pneumonia of the lower lobe of the
right lung.
Prescription of which drug of the listed MOST appropriate?
1) Salbutamol
2) Amoxiclav
3) Prednisolone
4) Seretid
5) Euphyllin
Program control on the topic:
Pneumonia
III variant

1. THE MOST common causative agent of pneumonia is ...


1) Pneumococcus
2) Mycobacterium tuberculosis
3) Cytomegalovirus
4) Anaerobic staphylococci
5) E. coli

2.A man 34 years old after suffering an acute respiratory illness noted
the appearance of cough with phlegm, chest pain, shortness of breath, an increase in body temperature to
39.4˚С, Objectively: in the lower lobe of the right lung there is a shortening of the percussion sound.
When
On auscultation, fine bubbling rales are heard in the same place.
Which of the following diagnoses is MOST likely?
1) Community-acquired pneumonia of the lower lobe of the right lung
2) Chronic bronchitis
3) Bronchial asthma
4) Chronic obstructive pulmonary disease
5) Exudative right-sided pleurisy

3. A 58-year-old woman consulted a therapist with complaints of coughing up sputum, increased


body temperature up to 380C, shortness of breath, severe sweating, weakness. In the anamnesis
for a long time she took cytostatics and hormones for rheumatoid arthritis.
On auscultation on the right below the angle of the scapula, fine bubbling rales are heard.
Which of the following diagnoses is MOST likely?
1) Community-acquired pneumonia
2) Aspiration pneumonia
3) Pneumonia in immunocompromised
4) Hospital-acquired pneumonia
5) Exudative pleurisy

4. A 75-year-old man is in intensive care after stomach surgery. In the evening he


the body temperature rose to 38 °, pains appeared in the right half of the chest,
aggravated by coughing. On auscultation to the right below the angle of the scapula in the patient
humid fine bubbling rales were heard, NPV 26 per minute.
Which of the following diagnoses is MOST likely?
1) Community-acquired pneumonia
2) Exudative pleurisy on the right
3) Immunodeficiency pneumonia
4) Hospital-acquired pneumonia
5) Dressler Syndrome

5. A 58-year-old woman complaining of coughing up sputum, shortness of breath, chest pains associated
with
breathing, increased body temperature, sweating, weakness. I got sick after
hypothermia. On auscultation, moist fine bubbling rales are heard in the lower parts of the right lung.
Which of the following examination methods
most reasonable for the delivery of a diagnosis?
1) Spirography
2) Radiography
3) Electrocardiography
4) Peak flowmetry
5) Pulse oxemitria
6. A 45-year-old woman with complaints of cough with mucopurulent sputum, increased
body temperature up to 39˚С, shortness of breath.
Which of the following diagnoses is MOST typical for this X-ray
organs of the chest?
1) Pneumonia of the lower lobe of the right lung
2) Lung cancer
3) COPD
4) Bronchial asthma
5) Pleurisy of the lower lobe of the right lung

7. Sputum analysis: amount - 50 ml, color - rusty, character - mucopurulent,


consistency - viscous, microscopy: epithelium - 1-2 in the field of view, leukocytes - 20-25 in
field of view, erythrocytes - 10-15 in the field of view, pneumococci ++, atypical cells - no,
mycobacterium tuberculosis - no.
Which of the following diagnoses is the MOST probable according to the analysis result
phlegm?
1) Acute bronchitis
2) Pneumonia
3) Bronchial asthma
4) Pleurisy
5) Lung cancer

8. A 54-year-old man after suffering an acute respiratory illness appeared with


cough with sputum, chest pain, shortness of breath, fever up to 39.4˚С, Objectively: NPV 26 per minute.
On auscultation in the lower sections of the right lung
wet fine bubbling rales are heard. In the analysis of blood leukocytes 20x10 9
/ l, rod of nuclear neutrophils - 8%, ESR 30 mm / hour, CRP (+++). On the radiograph
extensive infiltrative shadow in the lower lobe of the right lung.
What is the severity of the disease MOST consistent with the patient's data?
1) Light
2) Medium
3) Heavy
4) Medium
5) Very heavy

9. A 25-year-old man with a diagnosis of community-acquired pneumonia of the lower lobe of the right
lung
received outpatient treatment. On the 7th day, there are no complaints, the condition is satisfactory, with
auscultation of vesicular breathing, no wheezing.
What further tactics of patient management is MOST expedient?
1) Continue antibiotic therapy
2) Prescribe other antibiotics
3) Start to work
4) Control X-ray of the OGK on day 10
5) Prescribe inhaled corticosteroids

10. A 38-year-old woman consulted a doctor with complaints of coughing up phlegm, shortness of breath,
pain in
chest associated with breathing, fever, sweating, weakness. Got sick
after hypothermia. Objectively: on the right lower parts of the lungs, shortening
percussion sound. On auscultation, fine bubbling rales are heard in the same place.
Which of the following diagnoses is MOST likely?
1) Community-acquired pneumonia of the lower lobe of the right lung
2) Chronic obstructive bronchitis
3) Bronchial asthma
4) Chronic obstructive pulmonary disease
5) Aspiration pneumonia
Program control on the topic:

Bronchial asthma

I variant

1. A 30-year-old man. Complaints of shortness of breath, cough with the release of viscous white sputum.
Works as an operator at a poultry farm. Objectively: height-170 cm, weight-80 kg, BMI-27.7 kg / m2.
Normal skin color. During auscultation of the lungs, dry wheezing sounds are heard more in the upper
and middle parts.

Which of the following is THE MOST likely cause of shortness of breath?

1) Age and gender

2) Stress

3) Profession

+4) Smoking

5) Obesity

2. A 30-year-old man. Complaints of shortness of breath, cough with the release of viscous white sputum.
Works as a TV operator. Objectively: height-170 cm, weight-110 kg, BMI-38 kg / m2. Normal skin color.
During auscultation of the lungs, dry wheezing sounds are heard more in the upper and middle parts.
Preliminary diagnosis: Bronchial asthma.

Which of the following is THE MOST characteristic in the pathogenesis of the disease?

1) Emphysema

+2) Bronchospasm

3) Laryngospasm

4) Pneumosclerosis

5) Mucostasis

3. A 22-year-old woman. Complaints of attacks of suffocation, paroxysmal cough with difficult to


separate sputum, shortness of breath with minor physical exertion, nasal congestion. He has been
suffering from pollinosis since the age of 18. Objectively: the state of moderate severity. With percussion
of the lungs-a box sound, with auscultation, a large number of dry whistling wheezes are heard on
exhalation.

Which of these diagnoses is MOST likely?

1) Community-acquired lower lobe pneumonia


2) Chronic obstructive bronchitis

3) Nosocomial upper lobe pneumonia

4) Chronic obstructive pulmonary disease

+5) Bronchial asthma, persistent course

4. Female, 22 years old. Complaints of attacks of suffocation, paroxysmal cough with difficult to separate
sputum, shortness of breath with minor physical exertion, nasal congestion. He has been suffering from
pollinosis since the age of 18. Objectively: the state of moderate severity. With percussion of the lungs-a
box sound, with auscultation, a large number of dry whistling wheezes are heard on exhalation.

Which of the listed methods of examination IS the MOST reasonable for making a diagnosis?

+1) Spirography with a bronchodilator test

2) Fiber-optic bronchoscopy with biopsy

3) Electrocardiography, echocardiography

4) Computed tomography of the chest organs

5) R-graph of the chest organs in two projections

5. A 55-year-old man was diagnosed with bronchial asthma. Indicators of peak flowmetry of PSV <
60%, daily lability of PSV >30%.

What degree of severity of the disease is MOST consistent with the patient's data in accordance with the
clinical Protocol for the diagnosis and treatment of bronchial asthma (MH RK, 2019)?

1) Intermittent

2) Persistent light

+3) Persistent medium

4) Persistent heavy

5) Very heavy

6. The woman 34 years after the home cleaning you have shortness of breath, cough with mucous viscous
sputum, wheezing. The patient has a history of allergies to honey, herbs, and dust.

In the blood test, which indicator is most expected to increase?

1) Red blood cells

2) White blood cells

3) Platelets
4) Lymphocytes

+5) Eosinophils

7. A 39-year-old woman with complaints of coughing at night, nasal congestion, sneezing, and watery
eyes. Notes allergy to wormwood blooms. The data of spirometry: VEL-88%, FVC-65%, FEV1-68%,
FEV1/VEL-79%.

What violation of the function of external respiration is MOST consistent with spirometry data?

+1) Restrictive light degree

2) Restrictive expressed

3) Mixed

4) Mild obstructive

5) Moderately severe obstructive

8. A 32-year-old male. Went to the emergency department with complaints of suffocation and shortness
of breath. From the anamnesis: episodes like this have been observed before, but were stopped on their
own. This attack developed suddenly during the renovation of the apartment. Smokes 1 pack a day.
Objectively: the skin is moist, moderate cyanosis of the lips. During auscultation of the lungs, breathing is
weakened, dry wheezing.

Which of the following is THE MOST acceptable in this situation?

+1) Short-acting bronchodilators

2) Long-acting bronchodilators

3) Antibacterial drugs

4) Expectorants

5) Inhaled glucocorticosteroids

9. A 25-year-old woman was admitted to the emergency department with complaints of severe shortness
of breath with difficulty exhaling, reaching suffocation, poorly stopped by salbutamol. From the medical
history of the disease: she considers herself sick for about 2 years, when she began to notice the
appearance of shortness of breath with difficulty exhaling during walks, especially in the spring. I went to
the clinic and was diagnosed with bronchial asthma.

Which of these groups of drugs is the MOST appropriate for basic therapy?

1) Antibacterial therapy

2) Inhaled glucocorticosteroids

3) Short-acting bronchodilators
4) Detoxification therapy

+5) Long-acting bronchodilators

10. The woman of 40 years old diagnosed with bronchial asthma. Indicators of peak flowmetry of PSV ≥
80% of the norm, daily lability of PSV 20% - 30%.

What degree of severity of the disease is MOST consistent with the patient's data in accordance with the
clinical Protocol for the diagnosis and treatment of bronchial asthma (MH RK, 2019)?

1) Intermittent

+2) Persistent light

3) Persistent medium

4) Persistent heavy

5) Very heavy

Program control on the topic:

Bronchial asthma

II variant

1. Which of the following is THE MOST characteristic pathogenetic mechanism in bronchial asthma?+

+1) Hyperreactivity of the bronchi

2) The presence of exudate in the alveoli

3) Inflammatory reaction of the lungs to gas

4) Pulmonary hypertension

5) Destruction of alveolar walls

2. A 30-year-old male. Complaints of shortness of breath, shortness of breath, cough with the release of
viscous white sputum. Works as a TV operator. Objectively: height-170 cm, weight-110 kg, BMI-38 kg /
m2. Normal skin color. During auscultation of the lungs, dry wheezing sounds are heard more in the
upper and middle parts. Preliminary diagnosis: Bronchial asthma.

Which of the following is THE MOST characteristic in the pathogenesis of the disease?

1) Emphysema

+2) Bronchospasm
3) Laryngospasm

4) Pneumosclerosis

5) Mucostasis

3. Female, 22 years old. Complaints of attacks of suffocation, paroxysmal cough with difficult to separate
sputum, shortness of breath with minor physical exertion, nasal congestion. He has been suffering from
pollinosis since the age of 18. Preliminary diagnosis: Bronchial asthma.

Which of the listed laboratory methods of examination IS the MOST reasonable for making a diagnosis?

1) Bacterial sputum culture

+2) Immunoenzyme analysis for IgE

3) Determination of C reactive protein

4) Immunoenzyme analysis for the IgM

5) Blood on CYFRA

4. Female, 22 years old. Complaints of attacks of suffocation, paroxysmal cough with difficult to separate
sputum, shortness of breath with minor physical exertion, nasal congestion. He has been suffering from
pollinosis since the age of 18. The diagnosis was made: bronchial asthma, persistent course.

For daily self-monitoring of your condition, which of the following is most appropriate?

1) Blood pressure monitor

++2) Peak flow meter

3) Thermometer

4) Pulse oximeter

5) Blood glucose meter

5. The woman 34 years after the home cleaning you have shortness of breath, cough with mucous viscous
sputum, wheezing. The patient has a history of allergies to honey, herbs, and dust. In the General analysis
of sputum-eosinophils 7 %, the immunogram increased the level of IgE. The data of spirography: ZHEL-
85%, FEV1-50%, tiffno index-65%.

Which of these diagnoses is MOST likely?

1) Chronic bronchitis

2) Community-Acquired pneumonia

3) Hospital-acquired pneumonia

4) Bronchiectatic disease
++5) Bronchial asthma

6. A 39-year-old woman with complaints of coughing at night, nasal congestion, sneezing, and watery
eyes. Notes allergy to wormwood blooms.

In sputum analysis, which of these indicators is MOST likely to increase?

1) Epithelium

2) Eosinophils

3) White blood cells

4) Red blood cells

5) Pneumococci

7. A 30-year-old man was diagnosed with bronchial asthma. Spirometry with a bronchodilator test was
performed.

With a positive test, how many percent is THE MOST expected increase in the FEV1 indicator?

1) 2

2) 4

3) 5

4) 8

+5) 12

8. A 32-year-old male. He went to the emergency department with complaints of suffocation and
shortness of breath. From the anamnesis: episodes like this have been observed before, but were stopped
on their own. This attack developed suddenly during the renovation of the apartment. Smokes 1 pack a
day. Objectively: the skin is moist, moderate cyanosis of the lips. During auscultation of the lungs,
breathing is weakened, dry wheezing.

Which of the listed methods of drug administration is the MOST acceptable?

+1) Inhalation

2) Sublingual

3) Intramuscular

4) Infusion

5) Oral
9. A 45-year-old man was diagnosed with bronchial asthma. Indicators of peak flowmetry of PSV ≥ 80%
of the norm, daily lability of PSV less than 20%.

What degree of severity of the disease is MOST consistent with the patient's data in accordance with the
clinical Protocol for the diagnosis and treatment of bronchial asthma (MH RK, 2019)?

1) Intermittent

2) Persistent light

3) Persistent medium

4) Persistent heavy

5) Very heavy

10. A 55-year-old woman was diagnosed with bronchial asthma. Indicators of peak flowmetry of PSV 60-
80% of the norm, daily lability of PSV >30%.

What degree of severity of the disease is MOST consistent with the patient's data in accordance with the
clinical Protocol for the diagnosis and treatment of bronchial asthma (MH RK, 2019)?

1) Intermittent

2) Persistent light

3) Persistent medium

4) Persistent heavy

5) Very heavy

Program control on the topic:

Chronic obstructive pulmonary disease

I variant

1. A 60-year-old woman with a diagnosis of Chronic obstructive pulmonary disease. Worked as


a cook for many years. Which of the following is THE MOST likely cause of the disease?

++1) Lack of α1-antitrypsin

2) Bad habits

3) Age and gender of the patient

4) Occupational hazard

5) Duration of the disease lung disease

2. A 52-year-old man complained of a cough with difficult to separate sputum, shortness of


breath at rest, which increases with physical exertion. From anamnesis: the last 10 years notes a
constant cough in the morning and sputum discharge. Smokes for 30 years Objectively: barrel-
shaped chest, percussion-lung sound with a boxy tinge. During auscultation, the breath is
weakened, the exhalation is prolonged, and dry, scattered wheezes are heard. Which of the listed
methods of examination IS the MOST reasonable for making a diagnosis?

1) Bronchoscopy

2) Peak Flowmetry

3) Pulse oximetry

4) Chest x-Ray

5) Spirometry

3. A 45-year-old man with a diagnosis of Chronic obstructive pulmonary disease. When


performing spirometry, VEL – 85%, FEV1-60%, Tiffno index -68%. What severity DOES the
spirometry data most correspond to?

1) Light

+2) Medium

3) Medium heavy

4) Heavy

5) Extremely heavy

4. Male 62 years old, driver, smokes for 40 years. Complaints of shortness of breath, cough with
mucous sputum during the day. Objectively: with auscultation of the lungs, weakened breathing,
dry wheezing in all fields. Which of the following diagnoses is MOST likely?

+1) COPD

2) Bronchitis

3) Pneumonia

4) Idiopathic pulmonary fibrosis

5) Pleurisy

5. Male 62 years old, driver, smokes for 40 years. Complaints of shortness of breath, cough with
mucous sputum during the day. Objectively: with auscultation of the lungs, weakened breathing,
dry wheezing in all fields. Preliminary diagnosis: Chronic obstructive pulmonary disease.
Spirometry was performed. Which of the listed spirometry indicators is MOST likely to
decrease?

1) Vital capacity of lungs

2) MEF25
3) Maximum ventilation

4) Respiratory volume

+5) Tiffno Index

6. a 69-year-old man with complaints of shortness of breath at the slightest load. During
spirometry, the indicators of FVC – 39%, FEV1 – 23%, FEV1/FVC – 66% were determined.
What type of disorders of respiratory function is MOST consistent with the data obtained?

++1) very severe obstructive

2) severe restrictive

3) mild obstructive

4) severe obstructive

5) mixed type

7. In chronic obstructive pulmonary disease, which of the following complications is MOST


likely?

1) Nephritis

++2) Chronic pulmonary heart

3) Lung abscess

4) Metapneumonic pleurisy

5) Asthmatic status

8. Male 55 years old, smokes for about 30 years. Complaints of shortness of breath at the
slightest physical exertion, cough with mucopurulent sputum, sometimes with streaks of blood,
weight loss. The diagnosis was made: Chronic obstructive pulmonary disease, moderate severity,
exacerbation. According to the clinical Protocol "Chronic obstructive pulmonary disease", MH
RK 2016. which additional research method is MOST appropriate?

1) Electrocardiography

2) Computed tomography

+3) Spirometry

4) Peak flowmetry

5) Echocardiography

9. A 45-year-old man was diagnosed with Chronic obstructive pulmonary disease of moderate
severity. The appointment of any of the following groups of drugs is MOST appropriate?

+1) Bronchodilators
2) Vitamins

3) Spasmolytics

4) Antiagreganty

5) Antibiotics

10. Male 40 years old, smokes 1 pack a day. Complaints of coughing with mucus sputum in the
morning. When performing spirometry: FEV1-80%, Tiffno index-69%. Diagnosed with COPD,
mild. What is the most appropriate management strategy for the patient?

++1) Smoking cessation

2) Short –acting β2-agonists

3) Long-acting β2-agonists

4) M-cholinolytics

5) Inhaled corticosteroids

Program control on the topic:

Chronic obstructive pulmonary disease

II variant

1. A 58-year-old woman, a doctor. Complains of cough with mucosal sputum, shortness of


breath during exercise. He has been suffering from chronic obstructive pulmonary disease for 10
years. Do not smoke. Her sister was diagnosed with emphysema of the lungs. Which of the
following is THE MOST likely cause of the disease?

1) Age and gender of the patient

2) Occupational hazards

3) Air pollution

4) Duration of the disease

++5) Lack of α1-antitrypsin

2. A 52-year-old man complained of a cough with difficult to separate sputum, shortness of


breath at rest, which increases with physical exertion. From anamnesis: the last 10 years notes a
constant cough in the morning and sputum discharge. Smokes for 30 years Objectively: barrel-
shaped chest, percussion-lung sound with a boxy tinge. During auscultation, the breath is
weakened, the exhalation is prolonged, and dry, scattered wheezes are heard. Which of these
diagnoses is MOST likely?
1) Community-Acquired pneumonia

2) Chronic bronchitis

3) Pleurisy

4) Bronchial asthma

+5) Chronic obstructive pulmonary disease

3. A 52-year-old man with a diagnosis of Chronic obstructive pulmonary disease. When


performing spirometry, VEL – 65%, FEV1-45%, and the Tiffno index -55%. What severity
DOES the spirometry data most correspond to?

1) Light

2) Medium

3) Medium heavy

+4) Heavy

5) Extremely heavy

4. A 75-year-old man with a diagnosis of Chronic obstructive pulmonary disease. When


performing spirometry, VEL – 65%, FEV1-25%, Tiffno index -28%. What severity DOES the
spirometry data most correspond to?

1) Light

2) Medium

3) Medium heavy

4) Heavy

+5) Extremely heavy

5. Male 62 years old, driver, smokes for 40 years. Complaints of shortness of breath, cough with
mucous sputum during the day. Objectively: with auscultation of the lungs, weakened breathing,
dry whistling wheezes in all fields. Preliminary diagnosis: Chronic obstructive pulmonary
disease. Which of these survey methods will be most informative?

1) Peak flowmetry

++2) Spirometry

3) Bronchoscopy

4) Radioscopy

5) Electrocardiography
6. Male 62 years old, driver, smokes for about 40 years. Complaints of shortness of breath,
cough with mucous sputum during the day. Objectively: the thorax is barrel-shaped, percussion
lung sound with boxed shade. During auscultation, the breath is weakened, the exhalation is
prolonged, and dry, scattered wheezes are heard. He was diagnosed with Chronic obstructive
pulmonary disease. Which of the following changes in the chest x-ray is MOST typical for this
patient?

1) Homogeneous intense darkening with an oblique upper border

2) Intense infiltrative darkening of a rounded shape

3) A rounded cavity with a smooth internal contour

4) Focal-infiltrative changes with fuzzy, uneven contours

++5) Increased airiness of the lung tissue, low standing of the diaphragm

7. Which of the following spirometric indicators are MOST typical for patients with COPD
according to GOLD (2018)?

++1) FEV1 and FEV1/FVC

2) FVC and MEF 25%

3) MEF 50% and MEF 75%

4) VC and FVC

5) FVC and MVV

8. A man of 45 years, smoking for 6 years at 0.5 packs a day. The index of the smoker 3.
Diagnosed with Chronic obstructive pulmonary disease, mild severity, remission. According to
the clinical Protocol "Chronic obstructive pulmonary disease", MH RK 2016. What is the most
appropriate patient management strategy?

+1) Smoking cessation

2) Antibacterial therapy

3) Surgical treatment

4) Physical therapy

5) Oxygen therapy

9. A 40-year-old man smokes 1 pack a day for 10 years. Complaints of coughing with mucus
sputum in the morning. When performing spirometry: FEV1-80%, Tiffno index-69%.
Diagnosed with COPD, mild. What is the MOST appropriate management strategy for the
patient?
1) Antibiotics

+2) Short –acting β2-agonists

3) Long –acting β2 agonists

4) M-cholinolytics

5) Inhaled corticosteroids

10. A 67-year-old man is registered for COPD. He complained of a cough with mucopurulent
sputum, increased shortness of breath during exercise, and an increase in body temperature to
38°C. His condition is associated with hypothermia. When auscultation against the background
of weakened breathing with prolonged exhalation, scattered dry wheezes are heard from both
sides. Which of the following medications is most appropriate?

1) Salbutamol

+2) Amoxiclav

3) Prednisone

4) Teofillin

5) Amikacin

Program control on the topic:


Bronchiectasis
I variant

1. Select THE MOST likely definition that characterizes bronchiectatic lung disease:
1) Destruction of the walls of the alveoli
2) Hyperreactivity of the bronchi
3) Local suppuration process
+4) The presence of IRreversible changes in the bronchi
5) The Presence of intraalveolar exudation

2. A 15-year-old patient was admitted with complaints of cough with up to 200 ml of mucopurulent
sputum with an unpleasant smell, hemoptysis, fever up to 38.2 degrees Celsius , malaise, shortness of
breath. As a child, bronchitis often recurred and a cough with purulent sputum was bothering. Over the
past 5 years, has noted annual exacerbations. MOST likely diagnosis:
1) Lung Cancer
2) Chronic bronchitis
3) Bronchial asthma
+4) Bronchiectatic disease
5) Lung abscess

3. The MOST informative methods of diagnosis of bronchiectasis are:


+1) Chest x-Ray
2) Computed tomography of the lungs
3) Fibrobronchoscopy
4) Lung Scintigraphy
5) Ultrasound of the chest

4. The MOST likely complications of bronchiectasis include:


+1) Pulmonary hemorrhage + amyloidosis
2) Lung Cancer
3) Atelectasis of the lung lobe
4) Bronchial Adenocarcinoma
5) Pulmonary tuberculosis

5. The MOST informative objective clinical sign for the diagnosis of bronchiectatic disease is:
1) Shortening the percussion sound
2) Hard breathing
3) Dry whistling wheezes
4) Crepitating wheezes in the lower parts on both sides of the lungs
+5) Localized wet wheezes of different calibres

6. A 33-year-old woman, who smokes, has focal pneumonia with localization in S 9 for the third time in a
year. In the period between diseases, increased fatigue, cough, and sputum occur with the discharge of
purulent sputum up to 150 ml per day. Since childhood, he has suffered from bronchitis with frequent
exacerbations, which were accompanied by the separation of a large amount of sputum. MOST likely
diagnosis:
1) Cancer of the bronchus
2) Chronic bronchitis
3) Bronchial asthma
4) Bronchial Adenocarcinoma
+5) Bronchiectatic disease

7. A 14-year-old patient complained of shortness of breath, cough with up to 150 ml of mucopurulent


sputum with an unpleasant smell, hemoptysis, fever up to 38.6 degrees Celsius, general weakness. In
childhood, there were often bronchitis, a cough with the discharge of sputum of a purulent nature. Over
the past 6 years, there have been annual exacerbations. The MOST likely diagnosis:
+1) Bronchiectatic disease
2) Chronic bronchitis
3) Polycystic lung
4) Bronchial Adenocarcinoma
5) Lung Cancer

8. What is the most informative diagnostic method for a patient who complained of a cough with
mucopurulent sputum discharge up to 100 ml per day? In anamnesis: since childhood, the patient had
frequent episodes of coughing with a large amount of purulent sputum.
+1) Chest x-Ray
2) Computed tomography of the lungs
3) Fibrobronchoscopy
4) Lung Scintigraphy
5) Ultrasound of the chest

9. A 35-year-old patient came to the doctor complaining of a cough with a large amount of purulent
sputum, sometimes with a" full mouth " with an unpleasant putrid smell, lethargy, irritability, decreased
performance. He has a history of frequent bronchitis since childhood. When examined - fingers in the
form of "drumsticks", nails in the form of "watch glasses". On radiographs, against the background of an
enhanced pulmonary pattern, there is a cellular pattern in the lower lobe of the left lung. The mediastinal
organs are shifted to the left. What is THE most likely diagnosis?
1) Lung abscess
2) Chronic bronchitis
3) Chronic obstructive pulmonary disease
+4) Bronchiectatic disease
5) Bronchial Adenocarcinoma

10. A 34-year-old woman complained to the doctor about a cough with a purulent sputum discharge "full
mouth", sometimes up to 130 ml per day. In addition she was concerned about the weakness, fatigue,
subfebrilnaya temperature. The patient has been Smoking for 10 years . Since childhood, he has suffered
from bronchitis with frequent exacerbations, which were accompanied by the separation of a large
amount of sputum. Over the past year, she has had pneumonia three times with localization in the same
segment of the lung.
MOST likely diagnosis:
1) Lung abscess
+2) Bronchiectatic disease
3) Chronic obstructive pulmonary disease
4) Chronic bronchitis
5) Bronchial Adenocarcinoma

Program control on the topic:


Bronchiectasis
II variant
1. Which method of diagnosis is the MOST reliable for excluding bronchiectatic disease in a 25-year-old
patient who went to the doctor complaining of a cough with a large amount of purulent sputum.
+1) Chest x-Ray
2) Lung Scintigraphy
3) Layer-by-Layer tomography
4) Computed tomography of the lungs
5) Pleural ultrasound

2. The MOST likely risk factors for the development of bronchiectasis may be:
1) Lack of exercise
2) Violation of hemostasis
+3) Occupational hazards
4) Pneumonia at an early age
5) Age, gender, weight

3. In the pathogenesis of bronchiectatic disease, the MOST important role is played by:
1) Autoimmune mechanism
2) Inflammation of the bronchial mucosa
3) Obturation of the bronchi
+4) Lesions of the respiratory parts of the lungs
5) Changes in the bronchial mucosa

4. In bronchiectatic disease, the most characteristic changes on bronchoscopy:


1) Diffuse endobronchitis
2) Local endobronchitis
3) Hemorrhagic endobronchitis
4) Cat's eye symptom
5) Accumulation of mucus

5. In bronchiectatic disease, the MOST characteristic method of treatment is:


+1) Chest x-Ray
2) Bronchoscopy
3) Layer-by-Layer tomography
4) Computed tomography of the lungs
5) Pleural ultrasound

6. When auscultation of a patient with bronchiectatic disease, listen to:


1) Vesicular respiration
2) Amphoric sound
3) Decreased breath sounds
4) Dry wheezes
5) Wet multi-caliber wheezes

7. When studying the function of external respiration in a patient with bronchiectatic disease, determine:
1) Normal indicators
2) Minor deviations
3) Obstructive disorders
4) Restrictive infractions
5) Mixed infractions

8. Characteristic symptoms of bronchiectatic disease are:


1) Baldness symptom
2) The "Butterfly" symptom
+3) The "Drumsticks" symptom
4) Pasternatsky's symptom
5) "Cat's back" symptom

9. When bronchiectasis is localized in the lower lobe of the left lung, sputum is separated more easily:
1) in the evening
2) in a sitting position with a left tilt
3) in the position on the left side with the head end lowered
4) in a sitting position with an anterior tilt
++5) in the position on the right side with the head end lowered

10. In the classification of bronchiectatic disease, the MOST common forms are distinguished:
++1) Cylindrical
2) Spherical
3) Vitreous
4) Amygdala
5) Square

Program control on the topic:


Bronchiectasis
III variant
1. When bronchiectasis develops:

1) Acute inflammatory process

2) Chronic suppuration process

+3) Diffuse process

4) Reversible process

5) Destruction of lung tissue


2. Diagnosis of bronchiectatic disease is based on the identification of the following syndromes

1) Acute vascular insufficiency

2) Liver failure

++3) Purulent inflammation

4) Bronchial obstruction

5) Asthenovegetative

3. With an exacerbation of bronchiectatic disease, patients note:

1) Normal body temperature

2) Low-grade temperature

3) Periodic hemoptysis

4) Poor sputum separation

5) Sputum Separation by " full mouth»

4. In General blood analysis in patient with bronchiectasis disease is MOST often determine:

1) Erythrocytosis, slowing ESR

+2) Leukocytosis, acceleration of ESR

3) Eosinophilia, neutropenia

4) Leukocytopenia

5) Deceleration of ESR

5. With an exacerbation of bronchiectatic disease, sputum occurs:

1) Mucosa

2) Hemorrhagic

3) Vitreous

+4) Purulent, three-layer

5) With a putrid smell

6. In which position will be easier to move the sputum of a patient with localized bronchiectasis in the
lower lobe of the right lung?
1) in a horizontal position

2) in a sitting position with a left tilt

3) in the position on the left side with the head end lowered

4) in a sitting position with an anterior tilt

+5) in the position on the right side with the head end lowered

7. What kind of complications can be expected in a patient suffering from bronchiectasis disease for 35
years?

++1) Pulmonary bleeding

2) Lung Cancer

3) Atelectasis of the lung

4) Bronchial Adenocarcinoma

5) Chronic bronchitis

8. In the appearance of bronchiectasis MOST often plays the role of:

++1) Genetic inferiority of the bronchial tree

2) Hyperreactivity of the bronchi

3) Infection

4) Lack of alpha 1-antitrypsin

5) Carcinogens

9. Treatment of a patient with bronchiectatic disease includes:

1) Non-drug treatment

2) Sanation of the bronchial tree

3) Spa treatment

4) Rational mode of work and rest

++5) Complete nutrition, rich in protein and vitamins

10. On examination, the patient bronchiectasis disease is MOST often determine:

1) Shortening and deformation of fingers


++2) Thickening of the fingertips

3) The Brittle nails of the fingers

4) Delamination of finger nails

5) Osteolysis of the nail phalanges

Program control on the topic:


Idiopathic pulmonary fibrosis
I variant

1. Which research method IS most informative for idiopathic pulmonary fibrosis

1) Provocative test

2) Broncholytic test

3) Respiratory test

4) Metocholine test

5) Diffusion test

2. The radiograph for idiopathic pulmonary fibrosis MOST often reveals

1) Cavities in the lungs with an infiltrative wall

2) Multiple thin-walled cavities in the lungs

3) Enlarged and compacted lymph nodes in the basal zones

4) Massive infiltration in the lower parts of both lungs

++5) Symptoms of "frosted glass" and cellular lung

3. A 65-year-old man was diagnosed with idiopathic pulmonary fibrosis. The appointment of any of the
following groups of drugs is MOST appropriate?

1) Bronchodilators

2) Glucocorticosteroids

3) Antispasmodics

4) Antiplatelet Agents

5) Antibiotics

4. A 53-year-old housewife complains of progressive shortness of breath for 2-3 years, palpitations, dry
cough and chest pain that increase when trying to take a deep breath, weight loss, fever. Repeatedly and
without effect was treated with suspected pneumonia and chronic bronchitis. The x-ray shows increased
and deformed pulmonary pattern, interstitial fibrosis. The photo shows histological changes in the
subpleural parts of the lungs:
Which disease is MOST characteristic of this picture?
1) Silicosis
2) Sarcoidosis
3) Peripheral lung cancer
4) Disseminated tuberculosis
5) Idiopathic pulmonary fibrosis

5. A 52-year-old man with complaints of a wet cough, subfebrile fever against the background of
increasing shortness of breath over the past 3 months, in the lower parts of the lungs, crepitation is heard
in the "cellophane cod" type. Spirography: VEL-55%, FEV 160%, Tiffno index FEV1/VEL-110%,
MVL-35%. R-gram of the lungs-strengthening of the pulmonary pattern due to the fibrous component.
Which of these features is MOST likely to be detected by computed tomography?
1) Enlargement and compaction of lymph nodes in the root zones
2) Massive infiltration in the lower parts of both lungs
3) Infiltrative shadows in the area of the tops of the lungs
++4) Symptoms of "frosted glass" and cellular lung
5) Scattered focal shadows
6. A 62-year-old Man complains of a cough with copious mucopurulent sputum, moderate shortness of
breath. Objectively: pale skin, acrocyanosis of the nasolabial triangle, nails in the form of "watch
glasses", the chest is inactive when breathing; weakened vesicular breathing, shortened exhalation,
scattered dry wheezes. Palpable heartbeat, systolic murmur at the base of the xiphoid process, pasty shins.
Which of these changes is most likely to be detected during echocardiography?
1) Hypertrophy of the left ventricle and aortic insufficiency
2) Hypertrophy of the left ventricle and mitral insufficiency
3) Hyperkinesia of the posterior wall of the right ventricle
++4) Hypertrophy and dilatation of the right ventricle
5) Left ventricular hypertrophy and dilatation
7. Which of the following diagnoses is MOST typical for this chest x-ray?

1) Summany pleural effusion upper lobe of right lung

2) Aspiration pneumonia of the upper lobe of the right lung

+3) Idiopathic pulmonary fibrosis

4) Abscess of the upper lobe of the right lung

5) Nosocomial pneumonia of the upper lobe of the right lung

8. Which of the following diagnoses is MOST typical for this CT scan of the chest?

1) Summany pleural effusion upper lobe of right lung


2) Aspiration pneumonia of the upper lobe of the right lung
+3) Idiopathic pulmonary fibrosis
4) Abscess of the upper lobe of the right lung
5) Nosocomial pneumonia of the upper lobe of the right lung

9. For the definition of idiopathic pulmonary fibrosis is the MOST typical


1) acute inflammation of the bronchial mucosa
2) acute inflammation of the respiratory parts of the lungs
+3) chronic inflammation of the interstitial lung
4) a disease accompanied by progressive pulmonary fibrosis
5) lung disease characterized by attacks of suffocation

10. Which of the following is the most characteristic in the clinic of idiopathic pulmonary fibrosis?
+1) progressive shortness of breath
2) suffocation attacks
3) wet cough
4) severe sweating
5) chills

Program control on the topic:


Idiopathic pulmonary fibrosis
II variant

1. For the diagnosis of idiopathic pulmonary fibrosis MOST commonly uses the following methods
1) ultrasound of the chest
2) tomography of the mediastinum
3) fibrobronchoscopy
4) computed tomography
5) scintigraphy of the lungs

2. Two-sided fibrosis and bilateral changes of the "frosted glass" type in computed tomography are
MOST typical for:
1) idiopathic pulmonary fibrosis
2) bilateral pneumonia
3) miliary tuberculosis
4) amyloidosis of the lungs
5) sarcoidosis of the lungs

3. Which pathology is MOST characterized by gradually increasing shortness of breath, bilateral changes
in the lungs, restrictive type of respiratory disorders, the presence of disseminated mesh lung damage on
the x-ray, changes in the type of "frosted glass"according to computed tomography?

1) of the Pneumoconiosis

2) sarcoidosis of the lungs

3) Idiopathic pulmonary fibrosis

4) Interstitial pneumonia

5) Miliary tuberculosis of the lungs

4. A 43-year-old woman complained of fatigue, dry cough, shortness of breath, inability to take a deep
breath, weight loss. On examination: moderate cyanosis of the fingers are in the form of "drum sticks". In
the lungs-shortening of percussion sound in the lower parts on both sides, auscultation-tachypnea with
shortened inhalation and exhalation, crepitation in the interscapular region at the height of inspiration.
Tuberculosis was not detected.

What is the most likely preliminary diagnosis?

1) Idiopathic pulmonary fibrosis

2) Chronic obstructive pulmonary disease


3) Lingering bilateral pneumonia

4) Pneumoconiosis of the lungs

5) Lung Cancer

5. A 48-year-old patient complains of shortness of breath at rest, dry paroxysmal cough, palpitations,
weakness, fever, decreased appetite. Considers himself ill for about a year. X-ray revealed bilateral
fibrous changes, a symptom of "frosted glass". Antibacterial therapy did not have an effect – the
temperature increased, shortness of breath increased, and lost 6 kg in weight. Objectively: severe
condition, shortness of breath, acrocyanosis. BDD-32 in min., attenuated breathing and small-bubble
"crackling" wheezes are heard. In the blood: white blood cells-13 thousand, ESR-44 mm / h. What is the
MOST likely diagnosis?

1) disseminated pulmonary tuberculosis

2) exogenous allergic alveolitis

3) alveolar proteinosis

4) idiopathic pulmonary fibrosis

5) sarcoidosis of the lungs

6. A 41-year-old man was admitted with complaints of shortness of breath, a feeling of "lack of air"
during exercise, general weakness, weight loss, dry cough. Ill for 3-4 months. The clinic was diagnosed
with "pneumonia", but the appointment of antibiotics did not bring relief. Tuberculosis was not detected.
About: skin with moderate diffuse cyanosis. Fingers have the shape of "drumsticks", nails - "watch
glasses". BDD 24 V min.

What is the most likely preliminary diagnosis?

1) Lung Cancer

2) Sarcoidosis of the lungs

3) Focal bronchopneumonia

4) Chronic obstructive pulmonary disease

5) Idiopathic pulmonary fibrosis

7. THE MOST likely characteristic of fibrosing alveolitis:

1) Dullness, decreased breath sounds and bronhofoniya, displacement of the mediastinum to the opposite
side.

2) Bluntness, weakened breathing and bronchophonia, mediastinal displacement in the direction of


bluntness

3) Dullness with a tympanic sound, amphoric breathing, large-bubble wheezing


4) Inspiratory shortness of breath, decreased lung volume, crepitation.

5) Bluntness, hard breathing and bronchophonia, mediastinal displacement in the opposite direction

8. For idiopathic pulmonary fibrosis is the MOST typical all the signs except:

1) inspiratory dyspnea

2) expiratory dyspnea

3) temperature rise

4) crepitation

5) decreased breath sounds

9. A 47-year-old patient has been experiencing increasing shortness of breath over the past few months.
Anamnesis without features. Objectively: acrocyanosis, fingers in the form of drumsticks. In the lungs,
against the background of weakened breathing, there are no ringing wheezes, resembling crepitation.
There is an accent of the 2nd tone on the pulmonary artery. X-ray reticulation of the lung pattern, mainly
in the lower parts, the size of the heart is not changed, the cone of the pulmonary artery bulges out. ECG
shows signs of right ventricular hypertrophy. What is the most reasonable diagnosis?

1) Mitral stenosis

2) Idiopathic pulmonary fibrosis

3) COPD

4) Pneumonia

5) Hematogenically disseminated pulmonary tuberculosis

10. For idiopathic pulmonary fibrosis is the MOST typical

1) sharp start

2) the onset of the disease at the age of 40-70 years

3) severe sweating

4) joint damage приступы удушья

5) suffocation attacks

Mid-term examination in pulmonology


I variant
1. A 25-year-old man complains of weakness, dry cough for 2 days, pain behind the sternum,
increased body temperature to 37.0C, which appeared after hypothermia. In anamnesis: frequent
acute respiratory viral infection. Do not smoke. Determined by percussion a clear pulmonary
sound. Auscultatory auscultated breathing hard.
Which of these diagnoses is MOST likely?
1) Spontaneous pneumothorax
2) Exudative pleurisy
3) Lung Cancer
4) Community-Acquired pneumonia
5) Acute bronchitis

2. The MOST characteristic clinical feature of chronic bronchitis is


1) Cough for 3 months a year for 2 consecutive years
2) Cough that increases at night, attacks of suffocation
3) Chest pain associated with breathing, frequent and shallow breathing
4) Hemoptysis, sweating, weight loss, subfebrile temperature
5) Stunning chills, fever, severe symptoms of intoxication

3. Which of the following is THE MOST likely cause of community- acquired pneumonia?
1) Bronchoscopy
2) The post-operative period
3) Alcoholism
4) Diabetes mellitus
5) Hypothermia

4. A 68-year-old woman went to the doctor with complaints of cough with sputum, shortness of
breath, chest pain associated with breathing, fever, sweating, weakness. Got sick after
hypothermia. Objectively: on the right of the lower parts of the lungs, there is a shortened
percussion sound. During auscultation, small-bubble wheezes are also heard there.
Which of these diagnoses is MOST likely?
1) Community-Acquired pneumonia of the lower lobe of the right lung
2) Chronic obstructive bronchitis
3) Bronchial asthma
4) Chronic obstructive pulmonary disease
5) Aspiration pneumonia

5. A 24-year-old man complained of a cough with mucopurulent sputum, an increase in body


temperature to 38 C, shortness of breath. Abusing alcohol and smoking. Objectively on the hands
of traces of injections, the smell of vomit. During auscultation in the lower parts of the right
lung, weakened vesicular respiration.
Which of these diagnoses is MOST likely?
1) Community-Acquired pneumonia
+2) Aspiration pneumonia
3) Chronic bronchitis, exacerbation
4) Exudative pleurisy on the right
5) Lung Cancer

6. A 65-year-old man went to the therapist with complaints of a cough with mucopurulent
sputum, fever up to 38 C, shortness of breath. Which of the following descriptions is MOST
consistent with the presented chest x-ray?
1) Inflammatory infiltration in the lower lobe of the right lung
2) The destruction cavity in the lower lobe of the right lung
3) Focal formation in the lower lobe of the right lung
4) Two-way dissemination
5) Bilateral enlargement of the lung roots

7. A 29-year-old male. Complaints of severe pain in the right side of the chest, which increases
when inhaled, fever, pronounced weakness, dry cough. Got sick after hypothermia. Objectively:
the chest is of the usual shape, in the lower lobe of the right lung there is a dull percussive sound.
During auscultation in the lower parts of the right lung, breathing is sharply weakened.
Which of these diagnoses is MOST likely?
1) Community-Acquired lower lobe pneumonia
2) Aspiration pneumonia on the right
3) Dry pleurisy on the right
4) Exudative pleurisy on the right
5) Hospital-acquired pneumonia upper lobe

8. Which of the following is THE most likely cause of chronic obstructive pulmonary disease?
1) Hypothermia
2) Insolation
3) Obesity
4) Smoking
5) Alcohol

9. A 52-year-old man complained of a cough with difficult to separate sputum, shortness of


breath at rest, which increases with physical exertion. From anamnesis: the last 10 years notes a
constant cough in the morning and sputum discharge. Smokes for 30 years. Objectively: the
thorax is barrel-shaped, percussion pulmonary sound with boxed shade. During auscultation, the
breath is weakened, the exhalation is prolonged, and dry, scattered wheezes are heard.
Which of these diagnoses is MOST likely?
1) Community-Acquired pneumonia
2) Chronic bronchitis
3) Pleurisy
4) Bronchial asthma
5) Chronic obstructive pulmonary disease
10. Male 62 years old, driver, smokes for 40 years. Complaints of shortness of breath, cough
with mucous sputum during the day. Objectively: with auscultation of the lungs, weakened
breathing, dry wheezing in all fields.
Which of the following diagnoses is MOST likely?
1) COPD
2) Bronchitis
3) Pneumonia
4) Lung Cancer
5) Pleurisy

11. A 62-year-old male driver who has been smoking for about 40 years. Complaints of
shortness of breath, cough with mucous sputum during the day. Objectively: the chest is barrel-
shaped, percussion-lung sound with a boxy tinge. During auscultation, the breath is
weakened, the exhalation is prolonged, and dry, scattered wheezes are heard. He was diagnosed
with Chronic obstructive pulmonary disease.
Which of the following changes in chest radiography MOST typical for this patient?
1) homogeneous intense darkening with an oblique upper border
2) intensive infiltrative darkening of rounded shape
3) a rounded cavity with a smooth inner contour
4) focal-infiltrative changes with fuzzy, uneven contours
5) increased airiness of the lung tissue, low standing of the diaphragm

12. In chronic obstructive pulmonary disease, which of the following complications MOST
likely?
1) Nephrite
2) Chronic pulmonary heart
3) Lung abscess
4) Metapneumoviruses pleurisy
5) Asthmatic status

13. Which of the following is THE MOST likely cause of asthma?


1) Allergens
2) Smoking
3) Hypothermia
4) Alcohol
5) Insolation

14. A 22-year-old woman. Complaints of attacks of suffocation, paroxysmal cough with


difficult to separate sputum, shortness of breath with minor physical exertion, nasal congestion.
He has been suffering from pollinosis since the age of 18. Objectively: condition of moderate
severity. Diffuse "warm" cyanosis is noted. With percussion of the lungs-a box sound, with
auscultation, a large number of dry whistling wheezes are heard on exhalation.
Which of these diagnoses is MOST likely?
1) Community-Acquired lower lobe pneumonia
2) Chronic obstructive bronchitis
3) Nosocomial upper lobe pneumonia
4) Chronic obstructive pulmonary disease
5) Bronchial asthma, persistent course

15. Female, 22 years old. Complaints of attacks of suffocation, paroxysmal cough with
difficult to separate sputum, shortness of breath with minor physical exertion, nasal congestion.
He has been suffering from pollinosis since the age of 18. The diagnosis was made: Bronchial
asthma, persistent course.
For daily self-monitoring of your condition, which of the following is most appropriate?
1) blood pressure monitor
2) peak flow meter
3) thermometer
4) centimeter
5) blood glucose meter

16. A 39-year-old woman with complaints of coughing at night, nasal congestion, sneezing, and
lacrimation. Notes allergy to wormwood blooms. In sputum analysis, which of these indicators is
MOST likely to increase?
1) Epithelium
2) Eosinophils
3) White blood cells
4) Red blood cells
5) Pneumococci
17. A 45-year-old man was diagnosed with bronchial asthma. Peak flowmetry indicators PSV ≥
80% of the norm, the spread of PSV indicators is less than 20%.
What degree of severity of the disease is MOST consistent with the patient's data in
accordance with the clinical Protocol for the diagnosis and treatment of bronchial asthma (MH
RK, 2017)?
1) Intermittent
2) Persistent light
3) Persistent medium
4) Persistent heavy
5) Very heavy

18. Which of the following is THE MOST likely cause of chronic pulmonary heart:
1) hypertension of the large circulatory circle
2) hypertension of the small circle of blood circulation
3) portal hypertension
4) cerebral hypertension
5) renal hypertension

19. Which of the following indicators are most likely when making a diagnosis of a compensated
pulmonary heart?
1) Hypertrophy of the left heart and arterial hypertension
2) Hypertrophy of the right heart and pulmonary hypertension
3) Left ventricular heart failure
4) Right ventricular heart failure
5) Anasarka

20. What skin color changes are MOST likely to cause respiratory failure?
1) ≪Cold≫ cyanosis
2) ≪Warm≫ cyanosis
3) Acrocyanosis
4) Skin pallor
5) ≪Blush≫ on the cheeks
Mid-term examination in pulmonology
II variant

1. A 29-year-old woman complains of a sore and burning sensation behind the sternum, dry
cough, and weakness. In anamnesis: frequent acute respiratory viral infection. On examination,
the pallor of the skin is noted. Percussion is determined by the pulmonary sound. Auscultatory
auscultated hard breathing, no wheezing.
Which of these diagnoses is MOST likely?
1) Spontaneous pneumothorax
2) Acute bronchitis
3) Lung Cancer
4) Aspiration pneumonia
5) Dry pleurisy

2. A 48-year-old man complains of a cough with mucopurulent sputum during for a long time,
increased body temperature to 37.8C, weakness, malaise that appeared after hypothermia.
Objectively: cyanosis of the nasolabial triangle, thickening of the nails. Auscultated auscultation
hard breathing with krepitiruyuschie wheezing.
Which of these diagnoses is MOST likely?
1) Chronic bronchitis
2) Exudative pleurisy
3) Fibrosing alveolitis
4) Community-Acquired pneumonia
5) Acute bronchitis

3. Which of the following is THE most likely cause of aspiration pneumonia?


1) hormone therapy
2) the post-operative period
3) alcoholism
4) diabetes mellitus
5) hypothermia

4. A 30-year-old woman after severe hypothermia notes the appearance of pain in the right
half of the chest associated with breathing, an increase in body temperature to 39°C, chills,
coughing, sweating. Objectively: facial hyperemia, herpes labialis, the frequency of respiratory
movements- 26 per minute. In the lungs, the percussion sound is shortened to the right below the
angle of the scapula and wet, small-bubble wheezes are heard here. X-ray shows infiltration in
the lower lobe of the right lung.
Which of these diagnoses is MOST likely?
1) Community-Acquired pneumonia of the lower lobe of the right lung
2) Aspiration pneumonia of the lower lobe of the right lung
3) Immunodeficiency pneumonia
4) Nosocomial pneumonia of the lower lobe of the right lung
5) Chronic bronchitis, exacerbation

5. Male 75 years old is in intensive care after surgery on the stomach. In the evening, his body
temperature rose to 38°C, there were pains in the right half of his chest, which increased with
coughing. During auscultation to the right below the angle of the scapula, the patient had wet,
small-bubble wheezes, the frequency of respiratory movements 26 per minute.
Which of these diagnoses is MOST likely?
1) Community-Acquired pneumonia
2) Exudative pleurisy on the right
3) Immunodeficiency pneumonia
4) Nosocomial pneumonia
5) Dressler's Syndrome

6. A 68-year-old woman with complaints of cough with sputum, shortness of breath, chest pain
associated with breathing, fever, sweating, weakness. Got sick after hypothermia.
Which of the following diagnoses is MOST typical for this chest x-ray?

1) Community-Acquired lower lobe pneumonia


2) Chronic bronchitis
3) Bronchial asthma
4) Chronic obstructive pulmonary disease
5) Lung Cancer

7. A 54-year-old man with complaints of chest pain when inhaling deeply, shortness of breath,
and elevation body temperature, marked weakness, dry cough. Objectively: the chest is
of the usual shape, in the lower lobe of the left lung there is a dull percussive sound. During
auscultation in the lower parts of the left lung, breathing is sharply weakened.
Which of these diagnoses is MOST likely?
1) Community-Acquired lower lobe pneumonia
2) Aspiration pneumonia on the right
3) Dry pleurisy on the right
4) Exudative pleurisy on the left
5) Hospital-acquired pneumonia upper lobe

8. A 58-year-old woman, a doctor. Complains of cough with mucous sputum, shortness of breath
when exercise. He has been suffering from chronic obstructive pulmonary disease for 10 years.
Do not smoke. My sister was diagnosed with emphysema of the lungs.
Which of the following is THE MOST likely cause of the disease?
1) Age and gender of the patient
2) Occupational hazards
3) Air pollution
4) The duration of the disease
5) Lack of α1-antitrypsin

9. A 52-year-old man complained of a cough with difficult to separate sputum, shortness of


breath at rest, which increases with physical exertion. From anamnesis: the last 10 years notes a
constant cough in the morning and sputum discharge. Smokes for 30 years. Objectively: the
thorax is barrel-shaped, percussion pulmonary sound with boxed shade. During auscultation, the
breath is weakened, the exhalation is prolonged, and dry, scattered wheezes are heard.
Which of the listed methods of examination IS the MOST reasonable for making a diagnosis?
1) Spirometry
2) Peak flowmetry
3) Pulse oximetry
4) Radiography of the chest
5) Computed tomography

10. Male 62 years old, driver, smokes for 40 years. Complaints of shortness of breath, cough
with mucous sputum during the day. Objectively: with auscultation of the lungs, weakened
breathing, dry whistling wheezes all over the fields. Preliminary diagnosis: Chronic obstructive
pulmonary disease.
Which of these survey methods will be most informative?
1) Peak flowmetry
2) Spirometry
3) Bronchoscopy
4) Radioscopy
5) Electrocardiography

11. A 69-year-old man with complaints of shortness of breath at the slightest load. During
spirometry, the indicators of FVC – 39%, FEV1 – 23%, FEV1/FVC – 66% were determined.
What type of disorders of external respiration function correspond to the MOST
the data obtained?
1) very severe obstructive
2) severe restrictive
3) mild obstructive
4) severe obstructive
5) mixed type

12. A 45-year-old man was diagnosed with Chronic obstructive pulmonary disease,
mild severity, remission. According to the clinical Protocol "Chronic obstructive pulmonary
disease", MH RK 2013. What is the most appropriate patient management strategy?
1) Smoking cessation
2) Antibacterial therapy
3) Surgical treatment
4) Physical therapy
5) Oxygen therapy

13. A 28-year-old woman, after cleaning the house, notes difficulty breathing and lack of
air's. History: works as a salesman, food and pollen Allergy: mother has pollinosis.
Which of the following is the MOST likely cause?
1) Hypothermia
2) Heredity
3) Gender and age
4) Profession
5) Insolation

14. Female, 22 years old. Complaints of attacks of suffocation, paroxysmal cough with
difficult to separate sputum, shortness of breath with minor physical exertion, nasal congestion.
He has been suffering from pollinosis since the age of 18. Preliminary diagnosis: Bronchial
asthma.
Which of the listed laboratory methods of examination IS the MOST justified for diagnosis?
1) Bacterial sputum culture
2) Immunoenzyme analysis for IgE
3) Determination of C reactive protein
4) Immunoenzyme analysis for the IgM
5) Blood on CYFRA

15. A 34-year-old woman after home cleaning had shortness of breath, coughing attacks with
mucosal viscous sputum, wheezing. The patient has a history of allergies to honey, herbs, and
dust. In the General analysis of sputum-eosinophils 7 %, the immunogram increased the level of
IgE. The data of spirography: VC – 85%, FEV1 - 50%, the Tiffno index of 65%.
Which of these diagnoses is MOST likely?
1) Chronic bronchitis
2) Community-Acquired pneumonia
3) Hospital-acquired pneumonia
4) Bronchiectatic disease
5) Bronchial asthma

16. A 39-year-old woman with complaints of coughing at night, nasal congestion, sneezing, and
lacrimation. Notes allergy to wormwood blooms. The data of spirometry: VC -88%, FVC – 65%,
FEV1-68%, FEV1/VEL-79%.
What violation of the function of external respiration is MOST consistent with spirometry data?
1) Restrictive light degree
2) Restrictive expressed
3) Mixed
++4) Mild obstructive
5) Moderately severe obstructive

17. A 32-year-old male. He went to the emergency department with complaints of a choking
attack, dyspnea. From anamnesis: similar attacks were observed before, but were stopped
independently. This attack developed suddenly during the renovation of the apartment. Smokes 1
pack a day. Objectively: the skin is moist, moderate cyanosis of the lips. During auscultation of
the lungs, breathing is weakened, dry wheezing.
Which of the following is the MOST acceptable?
1) short-acting bronchodilators
2) long-acting bronchodilators
3) antibacterial drugs
4) expectorants
5) oxygen therapy, smoking cessation

18. The development of pulmonary hypertension in chronic pulmonary heart is MOST likely
within a few years:
1) Days
2) Weeks
3) Months
4) Hours
5) Years

19. Which of the following is THE MOST significant cause of narrowing of the pulmonary
vessels in chronic pulmonary heart disease?
1) Hemic hypoxia
2) Tissue hypoxia
3) Hypoxia of the brain
4) Hypoxia of the myocardium
5) Alveolar hypoxia

20. What are the most likely signs for decompensated pulmonary heart?
1) left ventricular failure
2) left heart failure
3) insufficiency of the mitral orifice
4) right ventricular failure
5) aortic valve insufficiency

You might also like